You are on page 1of 80

NURSING PRACTICE I: FOUNDATION OF NURSING 5.

Benner’s “Proficient” nurse level is different


PRACTICE from the other levels in nursing expertise in the
context of having:
CAUTION: Nursing is a profession. The nurse should a. the ability to organize and plan activities
have a background on the theories and foundation of b. having attained an advanced level of
nursing as it influenced what is nursing today. education
c. a holistic understanding and perception of
1. Nursing is the protection, promotion and the client
optimization of health and abilities, prevention d. intuitive and analytic ability in new
of illness and injury, alleviation of suffering situations
through the diagnosis and treatment of human
response and advocacy in the care of the SITUATION: The nurse has been asked to administer an
individuals, families, communities and the injection via Z TRACK technique. Questions 6 to 10 refer
population. This is the most accepted definition to this.
of nursing as defined by the:
a. PNA 6. The nurse prepares an IM injection for an adult
b. ANA client using the Z track technique. 4 ml of
c. Nightingale medication is to be administered to the client.
d. Herderson Which of the following site will you choose?
a. Deltoid
2. Advancement in nursing leads to the b. Rectus femoris
development of the Expanded Career Roles. c. Ventrogluteal
Which of the following is NOT an expanded d. Vastus lateralis
career role for nurses?
a. Nurse practitioner 7. In infants 1 year old and below, which of the
b. Nurse Researcher following is the site of choice for intramuscular
c. Clinical nurse specialist injection?
d. Nurse anaesthesiologist a. Deltoid
b. Rectos femoris
3. The Board of Nursing regulated the Nursing c. Ventrogluteal
profession in the Philippines and is responsible d. Vastus lateralis
for the maintenance of the quality of nursing in
the country. Powers and duties of the board of 8. In order to decrease discomfort in Z track
nursing are the following, EXCEPT: administration, which of the following is
a. Issue, suspend, revoke certificates of applicable?
registration a. Pierce the skin quickly and smoothly at a
b. Issue subpoena duces tecum, ad 90 degree angle
testification b. Inject the medication steadily at around 10
c. Open and close colleges of nursing minutes per milliliter
d. Supervise and regulate the practice of c. Pull back the plunger and aspirate for 1
nursing minute to make sure that the needle did
4. A nursing student or a beginning staff nurse not hit a blood vessel
who has not yet experienced enough real d. Pierce the skin slowly and carefully at a 90
situations to make judgments about them is in degree angle
what stage of Nursing Expertise?
a. Novice 9. After injection using Z Track technique, the
b. Newbie nurse should know that she needs to wait for a
c. Advanced Beginner few seconds before withdrawing the needle and
d. Competent this is to allow the medication to disperse into
the muscle tissue, thus decreasing the client’s
discomfort. How many seconds should the
nurse wait before withdrawing the needle?
a. 2 seconds
b. 5 seconds that the patient smokes and drinks coffee. When taking
c. 10 seconds the blood pressure of a client who recently smoked or
d. 15 seconds drank coffee, how long should the nurse wait before
taking the client’s blood pressure to accurate reading?
10. The rationale in using the Z track technique in a. 15 minutes
an intramuscular injection is: b. 30 minutes
a. It decreases the leakage of discolouring and c. 1 hour
irritating medication into the subcutaneous d. 5 minutes
tissues
b. It will allow a faster absorption of the 15. While the client has pulse oximeter on his
medication fingertip, you notice that the sunlight is shining
c. The Z track technique prevent irritation of on the area where the oximeter is. Your action
the muscle will be to:
d. It is much more convenient for the nurse a. Set and turn on the alarm of the oximeter
b. Do nothing since there is no identified
SITUATION: A Client was rushed to the emergency room problem
and you are his attending nurse. You are performing a c. Cover the fingertip sensor with a towel or
vital sign assessment. bedsheet
d. Change the location of the sensor every
11. All of the following are correct methods in four hours
assessment of the blood pressure EXCEPT:
a. Take the blood pressure reading on both 16. The nurse finds it necessary to recheck the
arms for comparison blood pressure reading. In case of such re
b. Listen to and identify the phases of assessment, the nurse should wait for a period
Korotkoff’s sound of?
c. Pump the cuff to around 50 mmHg above a. 15 seconds
the point where the pulse is obliterated b. 1 to 2 minutes
d. Observes procedures for injection control c. 30 minutes
d. 15 minutes
12. You attached a pulse oximeter to the client. You
know that the purpose is to: 17. If the arm is said to be elevated when taking the
a. Determine if the client’s hemoglobin level is blood pressure, it will create a:
low and if he needs blood transfusion a. False high reading
b. Check level of client’s tissue perfusion b. False low reading
c. Measure the efficacy of the client’s anti- c. True false reading
hypertensive medications d. Indeterminate
d. Detect oxygen saturation of arterial blood
before symptoms of hypoxemia develops 18. You are to assessed the temperature of the
client the next morning and found out that he
13. After a few hours in the Emergency Room, The ate ice cream. How many minutes should you
client is admitted to the ward with an order of wait before assessing the client’s oral
hourly monitoring of blood pressure. The nurse temperature?
finds that the cuff is too narrow and this will a. 10 minutes
cause the blood pressure reading to be: b. 20 minutes
a. Inconsistent c. 30 minutes
b. low systolic and high diastolic d. 15 minutes
c. higher than what the reading should be
d. lower than what the reading should be 19. When auscultating the client’s blood pressure
the nurse hears the following: From 150 mmHg
14. Through the client’s health history, you gather to 130 mmHg: Silence, Then: a thumping sound
continuing down to 100 mmHg; muffled sound
continuing down to 80 mmHg and then silence.
What is the client’s blood pressure? to lungs. This can be avoided by:
a. 130/80 a. Cleaning teeth and mouth with cotton swabs
b. 150/100 soaked with mouthwash to avoid rinsing the
c. 100/80 buccal cavity.
d. 150/100 b. swabbing the inside of the cheeks and lips,
tongue and gums with dry cotton swabs
20. In a client with a previous blood pressure of c. use fingers wrapped with wet cotton washcloth
130/80 4 hours ago, how long will it take to to rub inside the cheeks, tongue, lips and gums
release the blood pressure cuff to obtain an d. suctioning as needed while cleaning the buccal
accurate reading? cavity
a. 10-20 minutes
b. 30-45 minutes 25. Your client has difficulty of breathing and is
c. 1-1.5 minutes mouth breathing most of the time. This causes
d. 3-3.5 minutes dryness of the mouth with unpleasant odor.
Oral hygiene is recommended for the client and
SITUATION: Oral care is an important part of hygienic in addition, you will keep the mouth moistened
practices and promoting client comfort. by using:
a. salt solution
21. An elderly client, 84 years old, is unconscious. b. petroleum jelly
Assessment of the mouth reveals excessive c. water
dryness and presence of sores. Which of the d. metholated ointment
following is BEST to use for oral care?
a. lemon glycerine SITUATION – Ensuring safety before, during and after a
b. Mineral oil diagnostic procedure is an important responsibility of
c. hydrogen peroxide the nurse.
d. Normal saline solution
26. To help Fernan better tolerate the
22. When performing oral care to an unconscious bronchoscopy, you should instruct him to
client, which of the following is a special practice which of the following prior to the
consideration to prevent aspiration of fluids procedure?
into the lungs? a. Cleaning his fist every 2 minutes
a. Put the client on a sidelying position with b. Breathing in and out through the nose with
head of bed lowered his mouth open
b. Keep the client dry by placing towel under c. Tensing the shoulder muscles while lying on
the chin his back
c. Wash hands and observes appropriate d. Holding his breath periodically for 30
infection control seconds
d. Clean mouth with oral swabs in a careful
and an orderly progession 27. Following a bronchoscopy, which of the
following complains to Fernan should be noted
23. The advantages of oral care for a client include as a possible complication:
all of the following, EXCEPT: a. Nausea and vomiting
a. decreases bacteria in the mouth and teeth b. Shortness of breath and laryngeal stridor
b. reduces need to use commercial c. Blood tinged sputum and coughing
mouthwash which irritate the buccal d. Sore throat and hoarseness
mucusa
c. improves clients appearance and self 28. Immediately after bronchoscopy, you instructed
confidence Fernan to:
d. improves appetite and taste of food a. Exercise the neck muscles
24. A possible problem while providing oral care to b. Refrain from coughing and talking
unconscious client is the risk of fluid aspiration
c. Breathe deeply
d. Clear his throat 34. A nurse is reviewing the arterial blood gas
values of a client and notes that the ph is 7.31,
29. Thoracentesis may be performed for cytologic pco2 is 50 mmHg, and the bicarbonate is 27
study of pleural fluid. As a nurse your most mEq/L. The nurse concludes that which acid
important function during the procedure is to: base disturbance is present in this client?
a. Keep the sterile equipment from a. Respiratory acidosis
contamination b. Metabolic acidosis
b. Assist the physician c. Respiratory alkalosis
c. Open and close the three-way stopcock d. Metabolic alkalosis
d. Observe the patient’s vital signs
35. Allen’s test check the patency of the:
30. Right after thoracentesis, which of the following a. Ulnar artery
is most appropriate intervention? b. Carotid artery
a. Instruct the patient not to cough or deep c. Radial artery
breath for two hours d. Brachial artery
b. Observe for symptoms of tightness of
chest or bleeding Situation 6: Eileen, 45 years old is admitted to the
c. Place an ice pack to the puncture site hospital with a diagnostic of renal calculi. She is
d. Remove the dressing to check for bleeding experiencing severe flank pain, nauseated and with a
temperature of 39 OC.
Situation: Knowledge of the acid-base disturbance and
the functions of the electrolytes is necessary to 36. Given the above assessment data, the most
determine appropriate intervention and nursing actions. immediate goal of the nurse would be which of
the following?
31. A client with diabetes milletus has a blood a. Prevent urinary complication
glucose level of 644 mg/dL. The nurse interprets b. Maintains fluid and electrolytes
that his client is at most risk for the c. Alleviate pain
development of which type of acid-base d. Alleviating nausea
imbalance?
a. Respiratory acidosis 37. After IVP a renal stone was confirmed, a left
b. Respiratory alkalosis nephrectomy was done. Her post-operative
c. Metabolic acidosis order includes “daily urine specimen to be sent
d. Metabolic alkalosis to the laboratory”. Eileen has a foley catheter
attached to a urinary drainage system. How will
32. In a client in the health care clinic, arterial blood you collect the urine specimen?
gas analysis gives the following results: pH 7.48, a. Remove urine from drainage tube with
PCO2 32mmHg, PO2 94 mmHg, HCO3 24 mEq/L. sterile needle and syringe and empty urine
The nurse interprets that the client has which from the syringe into the specimen
acid base disturbance? container
a. Respiratory acidosis b. Empty a sample urine from the collecting
b. Metabolic acidosis bag into the specimen container
c. Respiratory alkalosis c. Disconnect the drainage tube from the
d. Metabolic alkalosis indwelling catheter and allow urine to flow
from catheter into the specimen container
33. A client has an order for ABG analysis on radial d. Disconnect the drainage from the collecting
artery specimens. The nurse ensures that which bag and allow the urine to flow to specimen
of the following has been performed or tested container
before the ABG specimens are drawn?
a. Guthrie test
b. Romberg’s test
c. Allen’s test
d. Weber’s test
38. Where would the nurse tape Ellen’s indwelling Regulation is secreted in the:
catheter in order to reduce urethal irritation? a. Thyroid gland
a. To the patient’s inner thigh b. Parathyroid gland
b. To the patient’s buttocks c. Hypothalamus
c. To the patient’s lower thigh d. Anterior pituitary gland
d. To the patient’s lower abdomen
45. While Parathormone, a hormone that negates
39. Which of the following menu is appropriate for the effect of calcitonin is secreted by the:
one with low sodium diet? a. Thyroid gland
a. Instant noodles, fresh fruits, and ice tea b. Parathyroid gland
b. Ham and cheese sandwich, fresh fruits anf c. Hypothalamus
vegetables d. Anterior pituitary gland
c. White chicken sandwich, vegetable salad
and tea Situation: The staff nurse supervisor requests all the
d. Canned soup, potato salad, and diet soda staff nurses to “brainstorm” and learn ways to instruct
diabetic clients on self-administration of insulin. She
40. How will you prevent ascending infection to wants to ensure that there are nurses available daily to
Eileen who has an indwelling catheter? do health education classes.
a. See to it that drainage tubing touches the
level of the urine 46. The plan of the nurse supervisor is an example
b. Changes the catheter every eight hours of
c. See to it that the drainage tubing does not a. In service education process
touch the level of the urine b. Efficient management of human resources
d. Clean catheter may be used since urethal c. Increasing human resources
meatus is not an sterile area d. Primary prevention

Situation: Hormones are secreted by the various glands 47. When Mrs. Guevarra, a nurse, delegates aspects
of the body. Basic knowledge of the endocrine system is of the clients care to the nurse-aide who is an
necessary. unlicensed staff, Mrs. Guevarra
a. makes the assignment to teach the staff
41. Somatocrinin or the Growth hormone releasing member
hormone is secreted by the: b. is assigning the responsibility to the aide
a. Hypothalamus but not the accountability for those tasks
b. Posterior pituitary gland c. does not have to supervise or evaluate the
c. Anterior pituitary gland aide
d. Thyroid gland d. most know how to perform task delegated

42. All of the following are secreted by the anterior 48. Connie, the new nurse, appears tired and
pituitary gland except: sluggish and lacks the enthusiasm she had six
a. Somtotropin/Growth hormone weeks ago when she started the job. The nurse
b. Thyroid stimulating hormone supervisor should
c. Follicle stimulating hormone a. Empathize with the nurse and listen to her
d. Gonadotropin hormone releasing hormone b. Tell her to take the day off
c. Discuss how she is adjusting to her new job
43. All of the following hormones are hormones d. Ask about her family life
secreted by the posterior pituitary gland except:
a. Vasopressin 49. Process of formal negotiations of working
b. Anti-diuretic hormone conditions between a group of registered
c. Oxytocin nurses and employer is
d. Growth hormone

44. Calcitonin, a hormone necessary for calcium


a. grievance d. It should disclose previous diagnostic,
b. arbitration prognosis, and alternative treatments
c. collective bargaining available for the client
d. strike
55. Delegation is the process of assigning tasks that
50. You are attending a certification on can be performed by a subordinate. The RN
cardiopulmonary resuscitation (CPR) offered should always be accountable and should not
and required by the hospital employing you. lose his accountability. Which of the following is
This is a role included in delegation?
a. professional course towards credit a. The RN must supervise all delegated tasks
b. in-service education b. After a task has been delegated, it is no
c. advance training longer a responsibility of the RN
d. continuing education c. The RN is responsible and accountable for
the delegated task in adjunct with the
Situation: As a nurse, you are aware that proper delegate
documentation in the patient chart is your responsibility d. Follow up with a delegated task is necessary
only if the assistive personnel is not
51. Which of the following is not a legally binding trustworthy
document but nevertheless very important in
the care of all patients in any health care Situation: When creating your lesson plan for
setting? cerebrovascular disease on STROKE. It is important to
a. Bill of rights as provided in the Philippine include the risk factors of stroke.
constitution
b. Scope of nursing practice as defined by RA 56. The most important risk factor is:
9173 a. Cigarette smoking
c. Board of nursing resolution adopting the b. binge drinking
code of ethics c. Hypertension
d. Patient’s bill of rigths d. heredity

52. A nurse gives a wrong medication to the client. 57. Part of your lesson plan is to talk about etiology
Another nurse employed by the same hospital or cause of stroke. The types of stroke based on
as a risk manager will expect to receive which of cause are the following EXCEPT:
the following communication? a. Embolic stroke
a. Incident report b. diabetic stroke
b. Nursing kardex c. Hemorrhagic stroke
c. Oral report d. thrombotic stroke
d. Complain report
58. Hemorrhagic stroke occurs suddenly usually
53. Performing procedure on a client in the absence when the person is active. All are causes of
of an informed consent can lead to which of the hemorrhage, EXCEPT:
following communication? a. phlebitis
a. Fraud b. damage to blood vessel
b. Harassment c. trauma
c. Assault and battery d. aneurysm
d. Breach of confidentiality
59. The nurse emphasizes that intravenous drug
54. Which of the following is the essence of abuse carries a high risk stroke. Which drug is
informed consent? closely linked to this?
a. It should have a durable power of attorney a. Amphetamines
b. It should have coverage from an insurance b. shabu
company c. Cocaine
c. It should respect the client’s freedom from d. Demerol
coercion
60. A participant in the Stroke class asks what is a d. Iron 75 mg/100 ml
risk factor of stroke. Your best response is:
a. “More red blood cells thicken blood and 65. Which of the following laboratory test result
make clots more possible.” indicate presence of an infectious process?
b. “Increase RBC count is linked to high a. Erythrocyte sedimentation rate (ESR) 12
cholesterol.” mm/hr
c. “More red blood cell increases hemoglobin b. White blood cells (WBC) 18,000/mm3
content.” c. Iron 90 g/100ml
d. “High RBC count increases blood pressure.” d. Neutrophils 67%

Situation: Recognition of normal values is vital in Situation: Pleural effusion is the accumulation of fluid in
assessment of clients with various disorders. the pleural space. Questions 66 to 70 refer to this.

61. A nurse is reviewing the laboratory test results 66. Which of the following is a finding that the
for a client with a diagnosis of severe nurse will be able to assess in a client with
dehydration. The nurse would expect the pleural effusion?
hematocrit level for this client to be which of a. Reduced or absent breath sound at the
the following? base of the lungs, dyspnea, tachypnea and
a. 60% shortness of breath
b. 47% b. Hypoxemia, hypercapnea and respiratory
c. 45% acidosis
d. 32% c. Noisy respiration, crackles, stridor, and
wheezing
62. A nurse is reviewing the electrolytes result of an d. Tracheal deviation towards the affected
assigned client and notes that the potassium side, increased fremitus and loud breath
level is 5.6 mEq/L. Which of the following would sounds.
the nurse expect to note on the ECG as a result
of this laboratory value? 67. Thoracentesis is performed to the client with
a. ST depression effusion. The nurse knows that the removal of
b. Prominent U wave fluid should be slow. Rapid removal of fluid in
c. Inverted T wave thoracentesis might cause:
d. Tall peaked T waves a. Pneumothorax
b. Cardiovascular collapse
63. A nurse is reviewing the electrolytes result of an c. Pleurisy or Pleuritis
assigned client and notes that the potassium d. Hypertension
level is 3.2 mEq/L. Which of the following would
the nurse expect to note on the ECG as a result 68. 3 Days after thoracentesis, the client again
of this laboratory value? exhibited respiratorydistress. The nurse will
a. U waves know that pleural effusion has reoccurred when
b. Elevated T waves she noticed a sharp stabbing pain during
c. Absent P waves inspiration. The physician ordered a close tube
d. Elevated ST Segment thoracotomy for the client. The nurse knows
that the primary function of the test tube is to:
64. Dorothy underwent diagnostic test and the a. Restore positive intrathoracic pressure
result of the blood examination are back. On b. Restore negative intrathoracic pressure
reviewing the results the nurse notices which of c. To visualize the intrathoracic content
the following as abnormal finding? d. As a method of air administration via
a. Neutrophils 60% ventilator
b. White blood cells (WBC) 9000/mm
c. Eryhrocyte sedimentation rate (ESR) is 39 69. The chest tube is functioning properly if:
mm/hr a. There is an oscillation
b. There is no bubbling in the drainage bottle
c. There is a continuous bubbling in the 75. This form of Health Insurance provides
waterseal comprehensive prepaid health services to
d. The sunction control bottle has a continuous enrollees for a fixed periodic payment.
bubbling a. Health Maintenance Organization
b. Medicare
70. In a client with pleural effusion, the nurse is c. Philippine Health Insurance Act
instructing appropriate breathing technique. d. Hospital Maintenance Organization
Which of the following is included in the
teaching? Situation: Nursing ethics is an important part of the
a. Breath normally nursing profession. As the ethical situation arises, so is
b. Hold the breath after each inspiration for 1 the need to have an accurate and ethical decision
full minute making.
c. Practice abdominal breathing
d. Inhale slowly and hold the breath for 3 to 5 76. The purpose of having a nurses’ code of ethics
seconds after each inhalation. is:
a. Delineate the scope and areas of nursing
Situation: Health care delivery system affect the health practice
status of every Filipino. As a Nurse, knowledge of this b. Identify nursing action recommended for
system is expected to ensure quality of life. specific health care situations
c. To help the public understand professional
71. When should rehabilitation commence? conduct expected of nurses
a. The day before discharge d. To define the roles and functions of the
b. When the patient desires health care givers, nurses, clients
c. Upon admission
d. 24 hours after discharge 77. The principles that govern right and proper
conduct of a person regarding life, biology and
72. What exemplified the preventive and promotive the health professionals is referred to as:
programs in the hospital? a. Morality
a. Hospital as a center to prevent and control b. Religion
infection c. Values
b. Program for smokers d. Bioethics
c. Program for alcoholics and drugs addicts
d. Hospital wellness center 78. A subject feeling about what is right or wrong is
said to be:
73. Which makes nursing dynamic? a. Morality
a. Every patient is a unique physical, b. Religion
emotional, social and spiritual being c. Values
b. The patient participate in the overall d. Bioethics
nursing care plan
c. Nursing practice is expanding in the light of 79. Values are said to be the enduring believe about
modern developments that takes place a worth of a person, ideas and belief. If values
d. The health status of the patient is are going to be a part of a research, this is
constantly changing and the nurse must be categorized under:
cognizant and responsive to these changes a. Qualitative
b. Experimental
74. Prevention in an important responsibility of the c. Quantitative
nurse in: d. Non Experimental
a. Hospitals
b. Community 80. The most important nursing responsibility
c. Workplace where ethical situations emerge in patient care
d. All of the above is to:
a. Act only when advised that the action is
ethically sound
b. Not take sides, remain neutral and fair 85. Base on the Code of Ethics for Filipino Nurses,
c. Assume that ethical questions are the What is regarded as the hallmark of nursing
responsibility of the health team responsibility and accountability?
d. Be accountable for his or her own actions a. Human rights of clients, regardless of creed
and gender
81. Why is there an ethical dilemma? b. The privilege of being a registered
a. the choices involved do not appear to be professional nurse
clearly right or wrong c. Health, being a fundamental right of every
b. a client’s legal right co-exist with nurse’s individual
profession obligation d. Accurate documentation of actions and
c. Decisions has to be made based on societal outcomes
norms.
d. decisions has to be made quickly, often Situation: As a profession, nursing is dynamic and its
under stressful conditions practice is directed by various theoretical models. To
demonstrate caring behavior, the nurse applies various
82. According to the code of ethics, which of the nursing models in providing quality nursing care.
following is the primary responsibility of the
nurse? 86. When you clean the bedside unit and regularly
a. Assist towards peaceful death attend to the personal hygiene of the patient as
b. Health is a fundamental right well as in washing your hands before and after a
c. Promotion of health, prevention of illness, procedure and in between patients, you indent
alleviation of suffering and restoration of to facilitate the body’s reparative processes.
health Which of the following nursing theory are you
d. Preservation of health at all cost applying in the above nursing action?
a. Hildegard Peplau
83. Which of the following is TRUE about the code b. Dorothea Orem
of ethics of Filipino Nurses, except: c. Virginia Henderson
a. The Philippine Nurses Association for being d. Florence Nightingale
the accredited professional organization
was given the privilege to formulate a 87. A communication skill is one of the important
Code of Ethics for Nurses which the Board competencies expected of a nurse.
of Nursing promulgated Interpersonal process is viewed as human to
b. Code for nurses was first formulated in human relationship. This statement is an
1982 published in the proceedings of the application of whose nursing model?
third Annual Convention of the PNA House a. Joyce Travelbee
of Delegates b. Martha Rogers
c. The present code utilized the code of Good c. Callista Roy
Governace for the Professions in the d. Imogene King
Phillippines
d. Certificates of Registration of registered 88. The statement “the health status of an
nurses may be revoked or suspended for individual is constantly changing and the nurse
violations of any provisions of the Code of must be cognizant and responsive to these
Ethics changes” best explains which of the following
facts about nursing?
84. Violation of the code of ethics might equate to a. Dynamic
the revocation of the nursing license. Who b. Client Centered
revokes the license? c. Holistic
a. PRC d. Art
b. PNA
c. DOH 89. Virginia Henderson professes that the goal of
d. BON nursing is to work independently with other
health care working assisting the patient to
gain independence as quickly as possible. Which of the include:
following nursing actions best demonstrates this theory a. Prescription of the doctor to the patient’s illness
in taking care of a 94 year old client with dementia who b. Plan of care for patient
is totally immobile? c. Patient’s perception of one’s illness
a. Feeds the patient, brushes his teeth, gives the d. Nursing problem and Nursing diagnosis
sponge bath
b. Supervise the watcher in rendering patient his 94. The medical records that are organized into
morning care separate section from doctors or nurses has
c. Put the patient in semi fowler’s position, set the more disadvantages than advantages. This is
over bed table so the patient can eat by himself, classifies as what type of recording?
brush his teeth and sponge himself a. POMR
d. Assist the patient to turn to his sides and allow b. Modified POMR
him to brush and feed himself only when he c. SOAPIE
feels ready d. SOMR

90. In the self-care deficit theory by Dorethea 95. Which of the following is the advantage of
Orem, nursing care becomes necessary when a SOMR or traditional recording?
patient is unable to fulfill his physiological, a. Increase efficiency in data gathering
psychological and social needs. A pregnant b. Reinforces the use of the nursing process
client needing prenatal check-up is classified as: c. The caregiver can easily locate proper
a. Wholly compensatory section for making charting entries
b. Supportive educative d. Enhances effective communication among
c. Partially compensatory health care team members.
d. Non compensatory
Situations: June is a 24 year old client with symptoms of
Situation: Documentation and reporting are just as dyspnea, absent breath sounds on the right lung and
important as providing patient care, As such, the nurse chest x ray revealed pleural effusion. The physician will
must be factual and accurate to ensure quality perform thoracentesis.
documentation and reporting.
96. Thoracentesis is usefulin treating all of the
91. Health care reports have different purposes. following pulmonary disorders except:
The availability of patients’ record to all health a. Hemothorax
team members demonstrates which of the b. Hydrothorax
following purposes: c. Tuberculosis
a. Legal documentation d. Empynea
b. Research
c. Education 97. Which of the following psychological
d. Vehicle for communication preparation is not relevant for him?
a. Telling him that the gauge of the needle
92. When a nurse commits medication error, she and anesthesia to be used
should accurately document client’s response b. Telling him to keep still during the
and her corresponding action. This is very procedure to facilitate the insertion of the
important for which of the following purposes: needle in the correct place
a. Reasearch c. Allow June to express his feelings and
b. Legal Documentation concerns
c. Nursing Audit d. Physician’s explanation on the purpose of
d. Vehicle for communication the procedure and how it will be done

93. POMR has been widely used in many teaching 98. Before thoracentesis, the legal consideration
hospitals. One of its unique features is SOAPIE you must check is:
charting. The P in SOAPIE charting should a. Consent is signed by the client
b. Medicine preparation is correct
c. Position of the client is correct
d. Consent is signed by relative and physician

99. As a nurse, you know that the position for June


before thoracentesis is:
a. Orthopedic
b. Low fowlers
c. Knee-chest
d. Sidelying position on the affected side

100. Which of the following anaesthetics


drug is used for thoracentesis?
a. Procaine 2%
b. Demerol 75 mg
c. Valium 250 mg
d. Phenobartbital 50 mg
NURSING PRACTICE II D. Follicle stimulating hormone

5. The following month, Mariah suspect she is


pregnant. Her urine is positive for Human
Situation: Mariah is a 31 year old lawyer who has been Chorionic Gonadotropin. Which structure
married for 6 months. She consults you for guidance in produces Hcg?
relation with her menstrual cycle and her desire to get A. Pituitary gland
pregnant. B. Trophoblastic cells of the embryo
C. Uterine deciduas
1. She wants to know the length of her menstrual D. Ovarian follicle
cycle. Her previous menstrual period is October
22 to 26. Her LMB is November 21. Which of the Situation: Mariah came back and she is now pregnant.
following number of days will be your correct
response? 6. At 5 month gestation, which of the following
A. 29 fetal development would probably be achieve?
B. 28 A. Fetal movement are felt Mariah
C. 30 B. Vernix caseosa covers the entire body
D. 31 C. Viable if delivered within this period
D. Braxton hicks contractions are observed.
2. You advised her to observe and record the signs
of ovulation. Which of the following signs will 7. The nurse palpates the abdomen of Mariah.
she likely note down? Now at 5 month gestation, what level of the
1. A 1 degree Fahrenheit rise in basal body abdomen can the fundic height be palpated?
temperature A. Symphysis pubis
2. Cervical mucus becomes copious and clear B. Midpoint between the umbilicus and the
3. One pound increase in weight xiphoid process
4. Mittelschmerz C. Midpoint between the symphysis pubis and
A. 1, 2 , 4 the umbilicus
B. 1, 2 , 3 D. Umbilicus
C. 2, 3, 4
D. 1, 3, 4 8. She worries about her small breasts, thinking
that she probably will not be able to breastfeed
3. You instruct Mariah to keep record of her basal her baby. Which of the following responses of
temperature every day, which of the following the nurse is correct?
instructions is incorrect? A. “The size of your breast will not affect your
A. If coitus has occurred; this should be lactation”
reflected in the chart B. “You can switch to bottle feeding”
B. It is best to have coitus on the evening C. “You can try to have exercise to increase
following a drop in BBT to become pregnant the size of your breast”
C. Temperature should be taken immediately D. “Manual expression of milk is possible”
after waking and before getting out of bed
D. BBT is lowest during the secretary phase 9. She tells the nurse that she does not take milk
regularly. She claims that she does not want to
4. She reports an increase in BBT on December gain too much weight during her pregnancy.
16. Which hormone brings about this change in Which of the following diagnosis is a priority?
her BBT? A. Potential self-esteem disturbance related to
A. Estrogen physiologic changes in pregnancy
B. Gonadotropin B. Ineffective individual coping related to
C. Progesterone physiologic changes in pregnancy
C. Fear related to the effects of pregnancy
D. Knowledge deficit regarding nutritional
requiremnts of pregnancies related to lack of 15. While talking with Susan, 2 new patients arrived
information sources and they are covered with large towels and the
nurse noticed that there are many cameraman
10. Which of the following interventions will likely and news people outside of the OPD. Upon
ensure compliance of Mariah? assessment the nurse noticed that both of them
A. Incorporate her food preferences that are are still nude and the male client’s penis is still
adequately nutritious in her meal plan. inside the female client’s vagina and the male
B. Consistently counsel toward optimum client said “I can’t pull it”. Vaginismus was your
nutritional intake first impression. You know that the
C. Respect her right to reject dietary psychological cause of Vaginismus is related to:
information if she chooses A. The male client inserted the penis too
D. Inform her of the adverse effect of deeply that it stimulates vaginal closure
inadequate nutrition to her fetus B. The penis was too large that is why the
vagina triggered its defense to attempt to
Situation: Susan is a patient in the clinic where you close it
work. She is inquiring about pregnancy. C. The vagina does not want to be penetrated
D. It is due to learning patterns of the female
11. Susan tell you she is worried because she client where she views sex as bad or sinful
develops breasts later than most of her friends.
Breast development is termed as: Situation: Overpopulation is one problem in the
A. Adrenarche Philippines that causes economic drain. Most Filipinos
B. Thelarche are against in legalizing abortion. As a nurse, Mastery of
C. Mamarche contraception is needed to contribute to the society and
D. Menarche economic growth.

12. Kevin, Susan’s husband tells you that he is 16. Supposed that Dana, 17 years old, tells you she
considering vasectomy after the birth of their wants to use fertility awareness method of
new child. Vasectomy involves the incision of contraception. How will she determine her
which organ? fertile days?
A. The testes A. She will notice that she feels hot, as if she
B. The epididymis has an elevated temperature.
C. The vas deferens B. She should assess whether her cervical
D. The scrotum mucus is thin, copious, clear and watery
C. She should monitor her emotions for
13. On examination, Susan has been found of sudden anger or crying
having a cystocele. A cystocele is: D. She should assess whether her breasts feel
A. A sebaceous cyst arising from the vulvar sensitive to cool air
fold
B. Protrusion of intestines into the vagina 17. Dana chooses to use COC as her family planning
C. Prolapse of the uterus into the vagina method. What is the danger sign of COC you
D. Herniation of the bladder into the vaginal would ask her to report?
wall A. A stuffy or runny nose
B. Slight weight gain
14. Susan typically has menstrual cycle of 34 days. C. Arthritis like symptoms
She told you she had coitus on days 8, 10, 15 D. Migraine headache
and 20 of her menstrual cycle. Which is the day
on which she is most likely to conceive? 18. Dana asks about subcutaneous implants and
A. 8th day she asks, how long will these implants be
B. Day 15 effective. Your best answer is:
C. 10th day A. One month
D. Day 20
B. Five years 23. Another client named Lilia is diagnosed as
C. Twelve months having endometriosis. This condition interferes
D. 10 years with fertility because:
A. Endometrial implants can block the
19. Dana ask about female condoms. Which of the fallopian tubes.
following is true with regards to female B. The uterine cervix becomes inflamed and
condoms? swollen
A. The hormone the condom releases might C. The ovaries stop producing adequate
cause mild weight gain estrogen
B. She should insert the condom before any D. Pressure on the pituitary leads to decreased
penile penetration FSH level
C. She should coat the condom with
spermicide before use 24. Lilia is scheduled to have a
D. Female condoms, unlike male condoms, are hysterosalphingogram. Which of the following
reusable instructions would you give her regarding this
procedure?
20. Dana has asked about GIFT procedure. What A. She will not be able to conceive for 3
makes her a good candidate for GIFT? months after the procedure
A. She has patent fallopian tubes, so fertilized B. The sonogram of the uterus will reveal any
ova can be implanted on them tumors present
B. She is RH negative, a necessary stipulation C. Many women experience mild bleeding as
to rule out RH incompatibility an after effect
C. She has normal uterus, so the sperm can be D. She may feel some cramping when the dye
injected through the cervix into it is inserted
D. Her husband is taking sildenafil, so all
sperms will be motile. 25. Lilia’s cousin on the other hand, knowing Nurse
Lorena’s specialization ask what artificial
Situation: Nurse Lorena is a family planning and insemination by donor entails. Which would be
Infertility Nurse Specialist and currently attends to your best answer if you were Nurse Lorena?
FAMILY PLANNING CLIENTS AND INFERTLE COUPLES. A. Donor sperm are introduced vaginally into
The following conditions pertain to meeting the nursing the uterus or cervix
needs of this particular population group. B. Donor sperm are injected intra
abdominally into each ovary
21. Dina, 17 years old, asks you how a tubal ligation C. Artificial sperm are injected vaginally to test
prevents pregnancy. Which would be the best tubal patency
answer? D. The husband’s sperm is administered
A. Prostaglandins released from the cut intravenously weekly
fallopian tubes can kill sperm
B. Sperm cannot enter the uterus because the Situation: You are assigned to take care of a group of
cervical entrance is blocked. patients across the lifespan.
C. Sperm can no longer reach the ova,
because the fallopian tubes are blocked 26. Pain in the elder persons requires careful
D. The ovary no longer releases ova as there is assessment because they:
nowhere for them to go. A. experienced reduce sensory perception
B. have increased sensory perception
22. The Dators are a couple undergoing testing for C. are expected to experience chronic pain
infertility. Infertility is said to exist when: D. had a decreased pain threshold
A. A woman has no uterus
B. A woman has no children 27. Administration of analgesics to the older
C. A couple has been trying to conceive for 1 persons requires careful patient assessment
year because older people:
D. A couple has wanted a child for 6 months A. are more sensitive to drugs
B. have increased hepatic, renal and D. Chronic poverty
gastrointestinal function 34. Which of the following signs and symptoms
C. have increased sensory perception would you most likely find when assessing and
D. mobilize drugs more rapidly infant with Arnold-Chiari malformation?
A. Weakness of the leg muscles, loss of
28. The elderly patient is at higher risk for urinary sensation in the legs, and restlessness
incontinence because of: B. Difficulty swallowing, diminished or absent
A. increased glomerular filtration gag reflex, respiratory distress
B. decreased bladder capacity C. Difficulty sleeping, hyper vigilant, and an
C. diuretic use arching of the back
D. dilated urethra D. Paradoxical irritability, diarrhea, and
vomiting
29. Which of the following is the MOST COMMON
sign of infection among the elderly? 35. A parent calls you and frantically reports that
A. decreased breath sounds with crackles her child has gotten into her famous ferrous
B. pain sulfate pills and ingested a number of these
C. fever pills. Her child is now vomiting, has bloody
D. change in mental status diarrhea, and is complaining of abdominal pain.
You will the mother to:
30. Priorities when caring for the elderly trauma A. Call emergency medical services (EMS) and
patient: get the child to the emergency room
A. circulation, airway, breathing B. Relax because these symptoms will pass
B. airway, breathing, disability (neurologic) and the child will be fine.
C. disability (neurologic), airway, breathing C. Administer syrup of ipecac
D. airway, breathing, circulation D. Call the poison control center

31. Preschoolers are able to see things from which 36. A client says she heard from a friend that you
of the following perspectives? stop having period once you are on the “pill”.
A. Their peers The most appropriate response would be:
B. Their own and their mother’s A. “The pill prevents the uterus from making
C. Their own and their caregivers’ such endometrial lining that is why periods
D. Only their own often be scant or skipped occasionally”.
B. “If your friend has missed her period, she
32. In conflict management, the win-win approach should stop taking pills and get a pregnancy
occurs when: test a soon as possible”.
A. There are two conflicts and the parties C. “The pill should cause a normal menstrual
agree to each other period every month. It sounds that your
B. Each party gives in on 50% of the friend not taking the pills properly”.
disagreements making up the conflict D. “Missed period can be very dangerous and
C. Both parties involved are committed to may lead to the formation of precancerous
solving the conflict cells”.
D. The conflict is settled out of court so the 37. The nurse assessing new born babies infants
legal system and the parties win. during their hospital stay after birth will notice
Which of the following symptoms as a primary
33. According to the social-interactional perspective manifestation of Hirschsprung’s disease?
of child abuse and neglect, four factors place A. A fine rush over the trunk
the family members at risk for abuse. These risk B. Failure to pass meconium during the first
factors are the family itself, the caregiver, the 24 to 48 hours after birth
child, and
A. The presence of a family crisis
B. The national emphasis on sex
C. Genetics
C. The skin turns yellow and then brown over release
the first 48 hours of life B. a woman is less able to keep the laceration
D. High-grade fever clean because of her fatigue
C. healing limited during pregnancy so these
38. A client is 7 months pregnant and has just been will not heal until after birth
diagnosed as having a partial placenta Previa. D. increased bleeding occur from uterine
She is stable and has minimal spotting and is pressure on leg veins.
being sent home. Which of these instructions to
the client may indicate a need for further 43. In working with the caregivers of a client with
teaching? an acute or chronic illness, the nurse would:
A. Maintain bed rest with bathroom A. Teach care daily and let the caregivers do a
privileges return demonstration just before discharge
B. Avoid intercourse for 3 days B. Difficulty swallowing, diminished or absent
C. Call if contractions occur gag reflex, and respiratory distress
D. Stay on left side as much as possible when C. Difficulty sleeping, hypervigillant, and an
lying down arching of the back
D. Paradoxical irritability, diarrhea and
39. A woman has been rushed to the hospital with vomiting
ruptured membrane. Which of the following
should the nurse check first? 44. Which of the following roles BEST exemplifies
A. Check for the presence of infection the expanded role of the nurse?
B. Assess for prolapse of the umbilical cord A. Circulating nurse in surgery
C. Check the maternal heart rate B. Medication nurse
D. Assess the color of the amniotic fluid C. Obstetrical nurse
D. Pediatric nurse practitioner
40. The nurse notes that the infant is wearing a
plastic-coated diaper. If a topical medication 45. According to DeRosa and Kochura’s (2006)
were to be prescribed and it were to go on the article entitled “Implement Culturally
stomachs or buttocks, the nurse would teach Competent Health care in your workplace”,
the caregivers to: cultures have different patterns of verbal and
A. Avoid covering the area of topical nonverbal communication. Which difference
medication with the diaper does?
B. Avoid the use of clothing on top of the A. NOT necessarily belong
diaper B. Personal behavior
C. Put the diaper on as usual C. Subject matter
D. Apply an icepack for 5 minutes to the D. Conversational style
outside of the diaper
41. Which of the following factors is most 46. You are the burse assigned to work with a child
important in determining the success of with acute glomerulonephritis. By following the
relationships used in delivering a nursing care? prescribed treatment regimen, the child
A. Type of illness of the client experiences a remission. You are now checking
B. Transference and counter transference to make sure the child does not have a relapse.
C. Effective communication Which finding would most lead you to the
D. Personality of the participants conclusion that a relapse is happening?
A. Elevated temperature, cough, sore throat,
42. Grace sustained a laceration on her leg from changing complete blood count (CBC) with
automobile accident. Why are lacerations of differential
lower extremities potentially more serious B. A urine dipstick measurement of 2+
among pregnant women than other? proteinuria or more for 3 days, or the child
A. lacerations can provoke allergic responses found to have 3-4+ proteinuria plus
due to gonadotropic hormone edema.
51. If a child with diarrhea registers two signs in the
C. The urine dipstick showing glucose in the yellow row in the IMCI chart, when can classify
urine for 3 days, extreme thirst, increase in the patient as:
urine output, and a moon face. A. Moderate dehydration
D. A temperature of 37.8 degrees (100 degrees B. Severe dehydration
F), flank pain, burning frequency, urgency C. Some dehydration
on voiding, and cloudy urine. D. No dehydration
47. The nurse is working with an adolescent who 52. Celeste has had diarrhea for 8 days. There is no
complains of being lonely and having a lack of blood in the stool, he is irritable, his eyes are
fulfillment in her life. The adolescent shies away sunken, and the nurse offers fluid to Celeste
from intimate relationships at times yet at other and the drinks eagerly. When the nurse pinched
times she appears promiscuous. The nurse will the abdomen it goes back slowly. How will you
likely work with this adolescent in which of the classify Celeste’s illness?
following areas? A. Moderate Dehydration
A. Isolation B. Severe dehydration
B. Lack of fulfillment C. Some dehydration
C. Loneliness D. No dehydration
D. Identity
53. A child who is 7 weeks has had a diarrhea for 14
48. The use of interpersonal decision making, days but has no sign of dehydration is classified
psychomotor skills, and application of as:
knowledge expected in the role of a license A. Persistent diarrhea
health care professional in the context of public B. Dysentery
health welfare and safety is an example of: C. Severe Dysentery
A. Delegation D. Severe Persistent Diarrhea
B. Responsibility
C. Supervision 54. The child with no dehydration needs home
D. Competence treatment. Which of the following is not
included in the rules for home treatment in this
49. The painful phenomenon known as “back labor” case?
occurs in a client whose fetus in what position? A. Forced Fluids
A. Brow position B. When to return
B. Breech position C. Give vitamin A supplement
C. Right Occipito- anterior position D. Feeding more
D. Left Occipito- posterior position
55. Fever as used in IMCI includes:
50. FOCUS methodology stands for: A. Axillary temperature of 37.5 or higher
A. Focus, Organize, Clarify, Understand and B. Rectal temperature of 38 or higher
Solution C. Feeling hot to touch
B. Focus, Opportunity, Continuous, Utilize, D. All of the above
Substantiate E. A and C only
C. Focus, Organize, Clarify, Understand,
Substantiate Situation: Prevention of Dengue is an important nursing
D. Focus, Opportunity, Continuous (process), responsibility and controlling its spread is a priority
Understand, Solution once outbreak has been observed.

Situation: The infant and child mortality rate in the low 56. An important role of the community health
middle income countries is ten times higher than nurse in the prevention and control of dengue
industrialized countries. In response to this, the WHO H-Fever includes:
and UNICEF launched the protocol integrated A. Advising the elimination of vectors by
management of Childhood Illness to reduce the keeping water containers covered
morbidity and mortality against childhood illnesses.
B. Conducting strong health education drives/ A. Identify the myths and misconceptions
campaign directed towards proper garbage prevailing in the community
disposal B. Identify the source of these myths and
C. Explaining to the individuals, families, misconceptions
group and community the nature of the C. Explain how and why these myths came
disease and its causation about
D. Practicing residual spraying with insecticides D. Select the appropriate IEC strategies to
correct them
57. Community health nurses should be alert in
observing a Dengue suspect. The following is 62. How many percent of measles are prevented by
not an indicator for hospitalization of H-Fever immunization at 9 months of age?
suspects? A. 80%
A. Marked anorexia, abdominal pain and B. 99%
vomiting C. 90%
B. Increasing hematocrit count D. 95%
C. Cough of 30 days
D. Persistent headache 63. After TT3 vaccination a mother is said to be
protected to tetanus by around:
58. The community health nurses’ primary concern A. 80%
in the immediate control of hemorrhage among B. 99%
patients with dengue is: C. 85%
A. Advising low fiber and Non- fat diet D. 90%
B. Providing warmth through light weight
covers 64. If ever convulsions occur after administering
C. Observing closely the patient for vital signs DPT, what should the nurse best suggest to the
leading to shock mother?
D. Keeping the patient at rest A. Do not continue DPT vaccination anymore
B. Advise mother to comeback after 1 week
59. Which of these signs may NOT be REGARDED as C. Give DT instead of DPT
a truly positive signs indicative of Dengue H- D. Give pertussis of the DPT and remove DT
Fever?
A. Prolonged bleeding time 65. These vaccines are given 3 doses at one month
B. Appearance of at least 20 petechiae within intervals:
1cm square A. DPT, BCG, TT
C. Steadily increasing hematocrit count B. OPV, HEP. B, DPT
D. Fall in the platelet count C. DPT, TT, OPV
D. Measles, OPV, DPT
60. Which of the following is the most important
treatment of patients with Dengue H-fever? Situation: with the increasing of documented cases of
A. Give aspirin for fever CANCER the best alternative to treatment still remains
B. Replacement of body fluids to be PREVENTION. The following conditions apply:
C. Avoid unnecessary movement of patient
D. Ice cap over the abdomen in case of melena 66. Which among the following is the primary focus
of prevention of cancer?
Situation: Health education and Health Promotion is an A. Elimination of conditions causing cancer
important part of nursing responsibility in the B. Diagnosis and treatment
community. Immunization is a form of health promotion C. Treatment at early age
that aims at preventing the common childhood D. Early detection
illnesses.
67. In the prevention and control of cancer, which
61. In correcting misconceptions and myths about of the following activities is the most important
certain diseases and their management, the
health worker should first: function of the community health nurse?
A. Conduct community assemblies D. 60 breaths per minute
B. Referral to cancer specialist those clients
with symptoms of cancer 73. Nina, the 2nd child has diarrhea for 5 days. There
C. Use the nine warning signs of cancer as is no blood in the stool. She is irritable, and her
parameters in our process of detection, eyes are sunken. The nurse offered fluid and the
control and treatment modalities child drinks eagerly. How would you classify
D. Teach woman about proper/correct Nina’s illness?
nutrition A. Some dehydration
B. Severe dehydration
68. Who among the following are recipients of the C. Dysentery
secondary level of care for cancer cases? D. No dehydration
A. Those under early case detection
B. Those under post case treatment 74. Nina’s treatment should include the following
C. Those scheduled for surgery EXCEPT:
D. Those undergoing treatment A. Reassess the child and classify him for
dehydration
69. Who among the following are recipients of the B. For infants under 6 months old who are not
tertiary level of care for cancer cases? breast feed, give 100-200 ml clean water as
A. Those under early treatment well during this period
B. Those under early detection C. Give in the health center the recommended
C. Those under supportive care amount of ORS for 4 hours
D. Those scheduled for surgery D. Do not give any other foods to the child for
home treatment
70. In community health nursing, despite the
availability and use of many equipment and 75. While on treatment, Nina 18 months old
devices to facilitate the job of the community weighted 18kgs. And her temperature
health nurse, the best tool any nurse should be registered at 37 degrees C. Her mother says she
will be prepared to apply is a scientific developed cough 3 days ago. Nina has no
approach. This approach ensures quality of care general danger signs. She has 45 breath/minute,
even at the community setting. This is nursing no chest in drawing, no stridor. How would you
parlance is nothing less than the: classify Nina’s manifestation?
A. Nursing diagnosis A. No Pneumonia
B. Nursing Research B. Pneumonia
C. Nursing Protocol C. Severe pneumonia
D. Nursing Process D. Bronchopneumonia

Situation: Two children were brought to you. One with 76. Carol is 15 months old and weight 5.5kgs. and it
Chest in drawing and the other had diarrhea. The is her initial visit. Her mother says that Carol is
following question apply: not eating well and unable to breastfeed, he is
no vomiting, has no convulsion and not
71. Using integrated Management and Childhood abnormally sleepy or difficult to awaken. Her
Illness (IMCI) approach, how would you classify temperature is 38.9 deg. C. Using the integrated
the 1st child? management of childhood illness or IMCI
A. Bronchopneumonia strategy, if you were the nurse in charge of
B. Severe pneumonia carol, how will you classify her illness?
C. No pneumonia: cough or cold A. a child at a general danger sign
D. Pneumonia B. severe pneumonia
C. very severe febrile disease
72. The 1st child who is 13 month has fast breathing D. severe malnutrition
using IMCI parameters he has:
A. 40 breaths per minute or more 77. Why are small for gestational age newborns at
B. 50 breaths per minute
C. 30 breaths per minute or more risk for difficulty maintaining body temperature?
A. their skin is more susceptible to conduction of iInform when to return to the health center
cold
B. they are preterm so are born relatively small in 83. Ms. Jordan, RN, believes that a patient should
size be treated as individual. This ethical principle
C. they do not have as many fast stored as other that the patient referred to:
infants A. beneficence
D. they are more active than usual so they throw B. respect for person
off comes C. nonmaleficence
D. autonomy
78. Oxytocin is administered to Rita to augment
labor. What are first symptoms of water 84. When patients cannot make decisions for
intoxication to observe for during this themselves, the nurse advocate relies on the
procedure? ethical principle of:
A. headache and vomiting A. justice and beneficence
B. a high choking voice B. beneficence and nonmaleficence
C. a swollen tender tongue C. fidelity and nonmaleficence
D. abdominal bleeding and pain D. fidelity and justice

79. Which of the following treatment shout NOT be 85. Being a community health nurse, you have the
considered if the child has severe dengue responsibility of participating in protecting the
Hemorrhagic Fever? health of people. Consider this situation:
A. use plan C is there is bleeding from the nose Vendors selling bread with their bare hands.
or gums They receive money with these hands. You do
B. give ORS if there is skin Petechiae, not see them washing their hands. What should
persistent vomiting, and positive tourniquet you say/do?
test A. “miss, may I get the bread myself because
C. give aspirin you have not washed your hands
D. prevent low blood sugar B. All of these
C. “Miss, it is better to use a pick up
80. In assessing the patient’s condition using the forceps/bread tong”
integrated management of Childhood illness D. “miss, your hands are dirty. Wash your
approach strategy, the first thing that a nurse hands first before getting the bread.
should do is to:
A. ask what are the child’s problem Situation: The following questions are refer to common
B. check for the four main symptoms clinical encounters experienced by an entry level nurse.
C. check the patients level of consciousness
D. check for the general danger signs 86. A female client ask the nurse about the use of a
cervical cap. Which statement is correct
81. A child with diarrhea is observed for the regarding the use of the cervical cap?
following EXCEPT: A. It may affect Pap smear results.
A. how long the child has diarrhea B. It does not need to be fitted by the
B. presence of blood in the stool physician.
C. skin petechiae C. It does not require the use of spermicide.
D. signs of dehydration D. It must be removed within 24 hours.

82. The child with no dehydration needs home 87. The major components of the communication
treatment. Which of the following is NOT process are:
included in the care for home management at A. Verbal, Written and nonverbal
this case?
A. give drugs every 4 hours

B. give the child more fluids B. Speaker, listener and reply


C. continue feeding the child
C. Facial expression, tone of voice, and 93. You are assigned in a private room of mike.
gestures Which procedure should be of outmost
D. Messages, sender, channel, receiver, and importance;
feedback A. Alcohol wash
B. Washing isolation
88. The extent of burns in children are normally C. Universal precaution
assessed and expressed in terms of: D. Proper nutrition
A. The amount of body surface that is
unburned 94. What primary health teaching would you give to
B. Percentages of total body surface area mike;
(TBSA) A. Daily exercise
C. How deep the deepest burns are B. Reverse isolation
D. The severity of the burns on a 1 to 5 burn C. Prevent infection
scale. D. Proper nutrition

89. The school nurse notices a child who is wearing 95. Exercise precaution must be taken to protect
old, dirty, poor fitting clothes; is always hungry; health worker dealing with the AIDS patients.
has no lunch money; and is always tired. When Which among these must be done as priority:
the nurse ask the boy his tiredness, he talks of A. Boil used syringe and needles
playing outside until midnight. The nurse will B. Use gloves when handling specimen
suspect that this child is: C. Label personal belonging
A. Being raised by a parent of low intelligence D. Avoid accidental wound
quotient (IQ)
B. An Orphan Situation: Michelle is a 6 years old preschooler. She was
C. A victim of child neglect reported by her sister to have a measles but she is at
D. The victim of poverty white home because of fever, upper respiratory
problem, and white sports in her mouth.
90. Which of the following indicates the type(s) of
accurate renal failure? 96. Rubeola is an Arabic term meaning red, the rash
A. Four types: hemorrhagic with and without appears on the skin in invasive stage prior to
clotting, and nonhemorrhagic with and eruption behind the ears. As a nurse, your
without clotting physical examination must determine
B. One type: acute complication especially:
C. Three types: prerenal, intrarenal, and A. Otitis media
postrenal B. Inflammatory conjunctiva
D. Two types: acute and subacute C. Bronchial pneumonia
D. Membranous laryngitis
Situation: Mike 16 y/o has been diagnosed to have
AIDS; he worked as entertainer in a cruise ship; 97. To render comfort, measure is one of the
priorities, which includes care of the skin, eyes,
91. Which method of transmission is common to ears, mouth, and nose. To clean the mouth your
contract AIDS? antiseptic solution is in some form of which one
A. Syringe and needles below?
B. Sexual contact A. Water
C. Body fluids B. Alkaline
D. Transfusion C. Sulfur
92. Causative organism in AIDS is one of the D. Salt
following:
A. Fungus 98. As a public health nurse, you teach mother and
B. Retrovirus family members the prevention of complication
C. Bacteria of measles. Which of the following should be

D. Parasites
Closely watched?
A. Temperature fails to drop
B. Inflammation of the nasophraynx
C. Inflammation of the conjunctiva
D. Ulcerative stomatitis

99. Source of infection of measles is secretion of


nosse and throat of infection person. Filterable
virus of measles is transmittedby:
A. Water supply
B. Food ingestion
C. Droplet
D. Sexual contact
100. Method of prevention is to avoid
exposure to an infection person. Nursing
responsibility for rehabilitation of patient
includes the provision of:
A. Terminal disinfection
B. Immunization
C. Injection of gamma globulin
D. Comfort measures
NURSING PRACTICE III
c. 50 days
d. 14 days
Situation: Leo lives in the squatter area. He goes to
nearby school. He helps his mother gather molasses Situation: as a nurse researcher you must have a very
after school. One day, he was absent because of fever, good understanding of the common terms of concept
malaise, anorexia, and abdominal discomfort. used in research.

1. Upon assessment, Leo was diagnosed to have 6. The information that an investigator collects
hepatitis a. Which mode of transmission has the from the subjects or participants in a research
infection agent taken? study is usually called;
A. Fecal-oral a. Hypothesis
B. Droplet b. Variable
C. Airborne c. Data
D. Sexual contact d. Concept

2. Which of the following is concurrent 7. Which of the following usually refers to the
disinfection in the case of Leo? independent variables in doing research
a. Investigation of contact a. Result
b. Sanitary disposal of faeces, urine and b. Output
blood c. Cause
c. Quarantine of the sick individual d. Effect
d. Removing all detachable objects in the
room, cleaning lighting and air duct surfaces 8. The recipients of experimental treatment is an
in the ceiling, and cleaning everything experimental design or the individuals to be
downward to the floor. observed in a non experimental design are
called;
3. Which of the following must be emphasized a. Setting
during mother’s class to Leo’s mother? b. Treatment
a. Administration of immunoglobulin to c. Subjects
families d. Sample
b. Thorough hand washing before and after
eating and toileting 9. The device or techniques an investigator
c. Use of attenuated vaccines employs to collect data is called;
d. Boiling of food especially meat a. Sample
b. Hypothesis
4. Disaster control should be undertaken when c. Instrument
there are 3 or more hepatitis A cases. Which of d. Concept
these measures is a priority?
a. Eliminate faecal contamination from foods 10. The use of another person’s ideas or wrong
b. Mass vaccination of uninfected individuals doings without giving appropriate credit result
c. Health promotion and education to from inaccurate or incomplete attribution of
families and communities about the materials to its sources. Which of the following
disease it’s cause and transmission is referred to when another person’s idea is
d. Mass administration of immunoglobulin inappropriate credited as one’s own;
a. Plagiarism
5. What is the average incubation period of b. Assumption
Hepatitis A? c. Quotation
a. 30 days d. Paraphrase
b. 60 days
Situation- Mrs. Pichay is admitted to your ward. The MD
ordered “prepare for thoracentesis this pm to remove
excess air from the pleural cavity.
11. Which of the following nursing responsibilities is a. Ease the patient to the floor To rule out
essential in Mrs. Pichay who will undergo pneumothorax
thoracentesis? b. Lift the patient and put him on the bed
a. Support and reassure client during the c. Insert a padded tongue depressor between
procedure his jaws
b. Ensure that informed consent has been d. Restraint patient’s body movement
signed
c. Determine if client has allergic reaction to 17. Mr. Santos is scheduled for CT SCAN for the
local anesthesia next day, noon time. Which of the followingis
d. Ascertain if chest x-rays and other tests n the correct preparation as instructed by the
have been prescribed and completed nurse?
a. Shampoo hair thoroughly to remove oil and
12. Mrs. Pichay who is for thoracentesis is assigned dirt
by the nurse to which of the following position? b. No special preparation is needed. Instruct
a. Trendelenburg position the patient to keep his head still and stead
b. Supine position c. Give a cleansing enema and give fluids until
c. Dorsal recumbent position 8 AM
d. Orthopneic position d. Shave scalp and securely attach electrodes
to it
13. During thoracentesis, which of the following
nursing intervention will be most crucial? 18. Mr. Santos is placed on seizure precaution.
a. Place patient in a quiet and cool room Which of the following would be
b. Maintain strict aseptic technique contraindicated?
c. Advise patient to sit perfectly still during a. Obtain his oral temperature
needle insertion until it has been b. Encourage to perform his own personal
withdrawn from the chest hygiene
d. Apply pressure over the puncture site as c. Allow him to wear his own clothing
soon as the needle is withdrawn d. Encourage him to be out of bed

14. To prevent leakage of fluid in the thoracic 19. Usually, how does the patient behave after his
cavity, how will you position the client after seizure has subsided?
thoracentesis? a. Most comfortable walking and moving
a. Place flat in the bed about.
b. Turn on the unaffected side b. Becomes restless and agitated
c. Turn on the affected side c. Sleeps for a period of time
d. On bed rest d. Say he is thirsty and hungry

15. Chest X-ray was ordered after thoracentesis. 20. Before, during and after seizure. The nurse
When your client asks what is the reason for knows that the patient is ALWAYS placed in
another chest x-ray, you will explain: what position?
a. To rule out pneumothorax a. Low fowler’s
b. To rule out any possible perforation b. Side lying
c. To decongest c. Modified trendelenburg
d. To rule out any foreign body d. Supine

Situation: A computer analyst, Mr. Ricardo J. Santos, 25 Situation: Mrs. Damian an immediate post op
was brought to the hospital for diagnostic workup after cholecystectomy and choledocholithotomy patient,
he had experienced seizure in his office. complained of severe pain at the wound site.

16. Just as the nurse was entering the room, the 21. Choledocholithotomy is:
patient who was sitting on his chair begins to a. The removal of the gallbladder
have a seizure. Which of the following must the b. The removal of the stones in the gallbladder
nurse do first? c. The removal of the stones in the
common bile duct b. Explain the proper use of PCA to alleviate
d. The removal of the stones in the kidney anxiety
c. Avoid overdosing to prevent
22. The simplest pain relieving technique is: dependence/tolerance
a. Distracton d. Monitor VS, more importantly RR
b. Deep breathing exercise
c. Taking aspirin 28. The client complained of abdominal distention
d. Positioning and pain. Your nursing intervention that can
alleviate pain is:
23. Which of the following statement on pain is a. Instruct client to go to sleep and relax
TRUE? b. Advice the client to close the lips and avoid
a. Culture and pain are not associated deep breathing and talking
b. Pain accompanies acute illness c. Offer hot and clear soup
c. Patient’s reaction to pain Varies d. Turn to sides frequently and avoid too
d. Pain produces the same reaction such as much talking
groaning and moaning
29. Surgical pain might be minimized by which
24. In pain assessment, which of the following nursing action in the O.R.
condition is a more reliable indicator? a. Skill of surgical team and lesser
a. Pain rating scale of 1 to 10 manipulation
b. Facial expression and gestures b. Appropriate preparation for the scheduled
c. Physiological responses procedure
d. Patient’s description of the pain sensation c. Use of modern technology in closing the
wound
d. Proper positioning and draping of clients
25. When a client complains pain, your initial
response is: 30. Inadequate anesthesia is said to be one of the
a. Record the description of pain common cause of pain both in intra and post op
b. Verbally acknowledge the pain patients. If general anesthesia is desired, it will
c. Refer the complaint to the doctor involve loss of consciousness. Which of the
d. Change to a more comfortable position following are the 2 general types of GA?
a. Epidural and Spinal
Situation: You are assigned at the surgical ward and b. Subarachnoid block and intravenous
Clients have been complaining of post pain at varying c. Inhalation and regional
Degrees. Pain as you know, is very subjective. d. Intravenous and inhalation

26. A one-day postoperative abdominal surgery Situation: Nurse’s attitude toward the pain influence
client has been complaining of severe throbbing The way they perceive and interact with clients in pain.
abdominal pain described as 9 in a 1-10 pain
rating. Your assessment reveals bowel sounds 31. Nurses should be aware that older adults are at
on all quadrants and the dressing is dry and risk of underrated pain. Nursing assessment and
intact. What nursing intervention would you management of pain should address the
take? following beliefs EXCEPT:
a. Medicate client as prescribed a. Older patient seldom tend to report
b. Encourage client to do imagery pain than the younger once
c. Encourage deep breathing and turning b. Pain is a sign of weakness
d. Call surgeon stat c. Older patients do not believe in
analgesics, they are tolerant
27. Pentoxidone 5 mg IV every 8 hours was d. Complaining of pain will lead to being
Prescribed for post abdominal pain. Which will labelled a ‘bad’ patient
be your priority nursing action? 32. Nurse should understand that when a client
a. Check abdominal dressing for possible responds favourable to a place, it is known as
swelling the ‘placebo effect’. Placebos do not indicate
Whether or not a client has:
a. Conscience Acting insulin first
b. Disease
c. Real pain 37. Janevi complains of nausea, vomiting,
d. Drug tolerance diaphoresis and headache. Which of the
following nursing intervention are you going to
33. You are the nurse in the pain clinic where you carry out first?
have client who has difficulty specifying the a. Withhold the client’s next insulin injection
location of pain. How can you assist such client? b. Test the client’s blood glucose level
a. The pain is vague c. Administer Tylenol as ordered
b. By charting- it hurts all over d. Offer fruit juice, gelatine and chicken
c. Identify the absence and presence of pain bouillon
d. As the client to point to the painful are by
just one finger 38. Janevi administered regular insulin at 7 am and
the nurse should instruct Jane to avoid
34. What symptom, more distressing than pain, exercising at around:
should the nurse monitor when giving opioids, a. 9 to 11 A.M
especially among elderly clients who are in b. Between 8 A.M to 9 A.M
pain? c. After 8 hours
a. Forgetfulness d. In the afternoon, after taking lunch
b. Drowsiness
c. Constipation 39. Janevi was brought at the emergency room
d. Allergic reactions like pruritis after four month because she fainted in her
clinic. The nurse should monitor which of the
35. Physical dependence occurs in anyone who following test to evaluate the overall
takes opioids over a period of time. What do therapeutic compliance of a diabetic patient?
you tell a mother of a ‘dependent’ when asked a. Glycosylated haemoglobin
for advice? b. Ketone levels
a. Start another drug and slowly lessen the c. Fasting blood glucose
opioid dosage d. Urine glucose level
b. Indulge in recreational outdoor activities
c. Isolate opioid dependent to a restful resort 40. Upon the assessment of Hba1c of Mrs. Segovia,
d. Instruct slow tapering of the drug dosage the nurse has been informed of a 9% Hba1c
and alleviate physical withdrawal result. In this case, she will teach the patient to:
symptoms a. Avoid infection
b. Prevent and recognize hyperglycaemia
Situation: The nurse is performing health education c. Take adequate food and nutrition
activities for Janevi Segovia, a 30 year old Dentist with d. Prevent and recognize hypoglycaemia
insulin dependent Diabetes Miletus.
41. The nurse is teaching plan of care for Jane with
36. Janevi is preparing a mixed dose of insulin. The regards to proper foot care. Which of the
nurse is satisfied with her performance when following should be included in the plan?
she: a. Soak feet in hot water
a. Draw insulin from the vial of clear insulin b. Avoid using mild soap on the feet
first c. Apply a moisturizing lotion to dry feet but
b. Draw insulin from the vial of the not between the toes
intermediate acting insulin first d. Always have a podiatrist to cut your toe
c. Fill both syringe with the prescribed insulin nails; never cut them yourself
dosage then shake the bottle vigorously
d. Withdraw the intermediate acting insulin 42. Another patient was brought to the emergency
first before withdrawing the short room in an unresponsive state and a diagnosis
of hyperglycaemia hyperosmolar nonketotic
syndrome is made. The nurse immediately
prepares to initiate which of the following of which of the following physiologic changes associated
anticipated physician’s order? with aging.
a. Endotracheal intubation a. Ineffective airway clearance
b. 100 unites of NPH insulin b. Decreased alveolar surfaced area
c. Intravenous infusion of normal saline c. Decreased anterior-posterior chest diameter
d. Intravenous infusion of sodium d. Hyperventilation
bicarbonate
47. The older patient is at higher risk for
43. Jane eventually developed DKA and is being incontinence because of:
treated in the emergency room. Which finding a. Dilated urethra
would the nurse expect to note as confirming b. increased glomerular filtration rate
this diagnosis? c. Diuretic use
a. Comatose state d. Decreased bladder capacity
b. Decreased urine output
c. Increased respiration and an increase in pH 48. Merle, age 86, is complaining of dizziness when
d. Elevated blood glucose level and low she stand up. This may indicate:
plasma bicarbonate level a. Dementia
b. Functional decline
44. The nurse teaches Jane to know the difference c. A visual problem
between hypoglycaemia and ketoacidosis. Jane d. Drug Toxicity
demonstrates understanding of the teaching by
stating the glucose will be taken if which of the 49. Cardiac ischemia in an older patient usually
following symptoms develops? produces:
a. Polyuria a. ST-T wave changes
b. Shakiness b. Chest pain radiating to the left arm
c. Blurred Vision c. Very high creatinine kinase level
d. Fruity breath odour d. Acute confusion

45. Jane has been scheduled to have FBS taken in 50. The most dependable sign of infection in the
the morning. The nurse tells Jane not to eat or older patient is:
drink after midnight. Prior to taking the blood a. Change in mental status
specimen the nurse noticed that Jane is holding b. Fever
a bottle of distilled water. The nurse asked Jane c. Pain
if she drink any, and she said “yes.” Which of d. Decreased breath sounds with crackles
the following is the best nursing action?
a. Administer syrup of ipecac to remove the Situation – In the OR, there are safety protocols that
distilled water from the stomach should be followed. The OR nurse should be well versed
b. Suction the stomach content using NGT with all these to safeguard the safety and quality of
prior to specimen collection patient delivery outcome.
c. Advice a physician to reschedule to
diagnostic examination next day 51. Which of the following should be given highest
d. Continue as usual and have the FBS priority when receiving patient in the OR?
analysis performed and specimen be taken a. Assess level of consciousness
b. Verify patient identification and informed
Situation: Elderly clients usually produce unusual signs consent
When it comes to different diseases. The ageing process c. Asses vital signs
is a complicated process and the nurse understand that d. Check for jewellery, gown, manicure, and
it is an inevitable fact and she must be prepared to care dentures
for the growing elderly population.
52. Surgeries like I and D (incision and drainage
46. Hypoxia may occur in the older patient because and debridement are relatively short
procedures but considered ‘dirty cases’ when
are these procedures best scheduled?
a. Last case The nurse knows that the temperature and time is set
b. In between cases to the optimum level to destroy not only the
c. According to availability of microorganism, but also the pores. Which of the
anaesthesiologist following is the ideal setting of the autoclave machine?
d. According to the surgeon’s preference a. 10,000 degree Celsius for 1 hour
53. Or nurses should be aware that maintaining the b. 5,000 degree Celsius for 30 minutes
client’s safety is the overall goal of nursing care c. 37 degree Celsius for 15 minutes
during the intraoperative phase. As the d. 121 degree Celsius for 15 minutes
circulating nurse, you make certain that
throughout the procedure… 58. It is important that before a nurse prepare the
a. The surgeon greets his client before material to be sterilized, a chemical indicator
induction of anaesthesia strip should be placed above the package,
b. The surgeon and anaesthesiologist are in preferably, Muslim sheet. What is the color of
tandem the striped produced after autoclaving?
c. Strap made of strong non-abrasive a. Black
materials are fastened securely around the b. Blue
joints of the knees and ankles and around c. Gray
the 2 hands around an arm board. d. Purple
d. Client is monitored throughout the surgery
by the assistant anaesthesiologist 59. Chemical indicators communicate that:
a. The items are sterile
54. Another nursing check that should not be b. That the items had undergone sterilization
missed before the induction of general process but not necessarily sterile
anaesthesia is: c. The items are disinfected
a. Check for presence underwear d. That the items had undergone disinfection
b. Check for presence dentures process but not necessarily disinfected
c. Check patient’s ID
d. Check baseline vital signs 60. If a nurse will sterilize a heat and moisture labile
instruments, it is according to AORN
55. Some lifetime habits and hobbies affect recommendation to use which of the following
postoperative respiratory function. If your client method of sterilization?
smokes 3 packs of cigarettes a day for the past a. Ethylene oxide gas
10 years, you will anticipate increased risk for: b. Autoclaving
a. Perioperative anxiety and stress c. Flash sterilizer
b. Delayed coagulation time d. Alcohol immersion
c. Delayed wound healing
d. Postoperative respiratory infection Situation: Nurses hold a variety of roles when providing
care to a perioperative patient.
Situation: Sterilization is the process of removing ALL
living microorganism. To be free of ALL loiving 61. Which of the following role would be the
microorganism is sterility. responsibility of the scrub nurse?
a. Assess the readiness of the client prior to
56. There are 3 general types of sterilization use in surgery
the hospital, which one is not included? b. Ensure that the airway is adequate
a. Steam sterilization c. Account for the number of sponges,
b. Physical sterilization needles, supplies, used during the surgical
c. Chemical sterilization procedure
d. Sterilization by boiling d. Evaluate the type of anaesthesia
appropriate for the surgical client
57. Autoclave or steam under pressure is the most
common method of sterilization in the hospital.
62. As a perioperative nurse, how can you best patients will need surgical amputation but there no
meet the safety need of the client after sterile surgical equipments. In this case, which of
administering preoperative narcotic? the following will the nurse expect?
a. Put side rails up and ask the client not to a. Equipments needed for surgery need not be
get out of bed sterilized if this is an emergency necessitating
b. Send the client to OR with the family life saving measures
c. Allow client to get up to go to the comfort b. Forwarding the trauma client to the nearest
room hospital that has available sterile equipment is
d. Obtain consent form appropriate
c. The nurse will need to sterilize the item before
63. It is the responsibility of the pre-op nurse to do using it to the client using the regular
skin prep for patients undergoing surgery. If hair sterilization setting at 121 degree Celsius in 15
at the operative site is not shaved, what should minutes
be done to make suturing easy and lessen d. In such cases, Flash sterilizer will be use at 132
chance of incision infection? degree Celsius in 3 minutes
a. Draped
b. Pulled 68. Tess, the PACU nurse, discovered that Malou,
c. Clipped who weighs 110lbs prior to surgery, is in severe
d. Shampooed pain 3 hrs after cholecystectomy. Upon
checking the chart, Malou found out that she
64. It is also the nurse’s function to determine has an order of Demerol 100 mg I.M prn for
when infection is developing in the surgical pain. Tess should verify the order with:
incision. The perioperative nurse should a. Nurse supervisor
observe for what signs of impending infection? b. Surgeon
a. Localized heat and redness c. Anaesthesiologist
b. Serosanguinous exudates and skin d. Intern on duty
blanching
c. Separation of the incision 69. Rosie, 57 who is diabetic is for debridement if
d. Blood clots and scar tissue are visible incision wound. When the circulating nurse
checked the present IV fluid, she found out that
65. Which of the following nursing interventions is there is no insulin incorporated as ordered.
done when examining the incision wound and What should the circulating nurse do?
changing the dressing? a. Double check the doctor’s order and call
a. Observe the dressing and type and odor the attending MD
of drainage if any b. Communicate with the ward nurse to verify
b. Get patient’s consent if insulin was incorporated or not
c. Wash hands c. Communicate with the client to verify if
d. Request the client to expose the incision insulin was incorporated
wound d. Incorporate insulin as ordered

Situation - The preoperative nurse collaborates with the 70. The documentation of all nursing activities
Client significant others, and healthcare providers. performed is legally and professionally vital.
Which of the following should NOT be included
66. To control environmental hazards in the Or, the in the patient’s chart?
nurse collaborates with the following a. Presence of prosthetoid devices such as
departments EXCEPT: dentures, artificial limbs hearing aid, etc.
a. Biomedical division b. Baseline physical , emotional, and
b. Infection control committee psychosocial data
c. Chaplaincy services c. Arguments between nurses and residents
d. Pathology department regarding treatments
d. Observed untoward signs and symptoms
67. An air crash occurred near the hospital leading and interventions including contaminant
to a surge of trauma patient. One of the last intervening factors
Situation- Team efforts is best demonstrated in the OR. Situation: Basic knowledge on Intravenous solution is
necessary for care of clients with problems with fluids
71. If you are the nurse in charge for scheduling and electrolytes
surgical cases, what important information do
you need to ask the surgeon? 76. A client involved in a motor vehicle crash
a. Who is your internist presents to the emergency department with
b. Who is your assistant and severe internal bleeding. The client is severely
anaesthesiologist, what is your preferred hypotensive and unresponsive. The nurse
time and type of surgery? anticipates which of the following intravenous
c. Who are your anaesthesiologist, internist, solutions will most likely be prescribed to
and assistant increase intravascular volume, replace
d. Who is your anaesthesiologist immediately blood loss and increablood
pressure?
72. In the OR, the nursing tandem for every surgery a. 0.45% sodium chloride
is: b. 0.33% sodium chloride
a. Instrument technician and circulating nurse c. Normal saline solution
b. Nurse anaesthetist, nurse assistant, and d. Lactated ringer’s solution
instrument technician
c. Scrub nurse, nurse anaesthetist 77. The physician orders the nurse to prepare an
d. Scrub and circulating nurse isotonic solution. Which of the following IV
solution would the nurse expect the intern to
73. While team effort is needed in the OR for prescribe?
efficient and quality patient care delivery, we a. 5% dextrose in water
should limit the number of people in the room b. 0.45% sodium chloride
for infection control. Who comprise this team? c. 10% dextrose in water
a. Surgeon, anaesthesiologist, scrub nurse, d. 5% dextrose in 0.9% sodium chloride
radiologist, orderly
b. Surgeon, assistants, scrub nurse, circulating 78. The nurse is making initial rounds on the
nurse, anaesthesiologist nursing unit to assess the condition of assigned
c. Surgeon, assistant surgeon, clients. The nurse notes that the client’s IV site
anaesthesiologist, scrub nurse, pathologist is cool, pale and swollen and the solution is not
d. Surgeon, assistant surgeon, infusing. The nurse concludes that which of the
anaesthesiologist, intern, scrub nurse following complications has been experienced
by the client?
74. Who usually act as an important part of the OR a. Infection
personnel by getting the wheelchair or b. Phlebitis
stretcher, and pushing/pulling them towards c. Infiltration
the operating room? d. Thrombophlelibitis
a. Orderly/clerk
b. Nurse supervisor 79. A nurse reviews the clients electrolyte
c. Circulating nurse laboratory report and notes that the potassium
d. Anaesthesiologist level is 3.2 mEq/L. Which of the following
complications has been experienced by the
75. The breakdown in teamwork is often times a client?
failure in: a. U waves
a. Electricity b. Absend P waves
b. Inadequate supply c. Elevated T waves
c. Leg work d. Elevated ST segment
d. Communication
80. One patient had a “runaway’ IV of 50%
dextrose. To prevent temporary excess of
insulin or transient hyperinsulin reaction what
solution you prepare in anticipation of doctor’s
Order? 86. As an OR nurse, what are your foremost
a. Any IV solution available to KVO considerations for selecting chemical agents for
b. Isotonic solution disinfection?
c. Hypertonic solution a. Material compatibility and efficiency
d. Hypotonic solution b. Odor and availability
c. Cost and duration of disinfection process
81. An informed consent is required for: d. Duration of disinfection and efficiency
a. Closed reduction of a fracture
b. Irrigation of the external ear canal 87. Before you use a disinfected instrument it is
c. Insertion of intravenous catheter essential that you:
d. Urethral catheterization a. Rinse with tap water followed by alcohol
b. Wrap the instrument with sterile water
82. Which of the following is not true with regards c. Dry the instrument thoroughly
to be informed consent? d. Rinse with sterile water
a. It should describe different treatment
alternatives 88. You have a critical heat labile instrument to
b. It should contain a thorough and detailed sterilize and are considering to use high level
explanation to the procedure to be done disinfectant. What should you do?
c. It should describe the clients diagnosis a. Cover the soaking vessel to contain the
d. It should give an explanation of the client’s vapour
prognosis b. Double the amount of high level
disinfectant
83. You know that the hallmark of nursing c. Test the potency of a high level disinfectant
accountability is the: d. Prolong the exposure time according to
a. Accurate documentation and reporting manufacturer’s direction
b. Admitting your mistakes
c. Filling an incident report 89. To achieve sterilization using disinfectants,
d. Reporting a medication error which of the following is used?
a. Low level disinfectants immersion in 24
84. A nurse is assigned to care for a group of clients. hours
On review of the client’s medical records, the b. Intermediate level disinfectants immersion
nurse determines that which client is at risk for in 12 hours
excess fluid volume? c. High level disinfectants immersion in 1 hour
a. The client taking diuretics d. High level disinfectant immersion in 10
b. The client with renal failure hours
c. The client with an ileostomy
d. The client who requires gastrointestinal 90. Bronchoscope, thermometer, Endoscope, ET
suctioning tube, Cytoscope are all BEST sterilized using
which of the following?
85. A nurse is assigned to care for a group of clients. a. Autoclaving at 121 degree Celsius in 125
On review of the client’s medical records, the minutes
nurse determines that which client is at risk for b. Flash sterilizer at 132 degree Celsius in 3
deficient fluid volume? minutes
a. A client with colostomy c. Ethylene oxide gas aeration for 20 hours
b. A client with congestive heart failure d. 2% Glutaraldehyde immersion for 10 hours
c. A client with decreased kidney function
d. A direct receiving frequent wound irrigation Situation: The OR is divided into three zones to control
traffic flow and contamination.
Situation: as a perioperative nurse, you are aware of the
correct processing methods for preparing instruments
and other devices for patient use to prevent infection.
91. What OR attires are worn in the restricted area? 97. 2 organizations endorsed that sterility are
a. Scrub suites, OR shoes, head cap affected by factors other than the time itself,
b. Head cap, scrub suit, mask, OR shoes these are:
c. Mask, OR shoes, scrub suit a. The PNA, and the PRC
d. Cap, mask, gloves, shoes b. AORN and JCAHO
c. ORNAP and MCNAP
92. Nursing intervention for a patient on low dose d. MMDA and DILG
IV insulin therapy includes the following
EXCEPT: 98. All of these factors affect the sterility of the OR
a. Elevation of serum ketones to monitor equipment, these are the following except:
ketosis a. The material used for packaging
b. Vital signs including BP b. The handling of the materials as well as its
c. Estimate serum potassium transport
d. Elevation of blood glucose levels c. Storage
d. The chemical or process used in sterilizing
93. The doctor ordered to incorporate 1000”u” the material
insulin to the remaining on –going IV. The
strength is 500/ml. how much should you 99. When you say sterile, it means:
incorporate into the IV solution? a. The material is clean
a. 10ml b. The material as well as the equipment are
b. 0.5ml sterilized and had undergone a rigorous
c. 2 ml sterilization process
d. 5ml c. There is black stripe on the paper indicator
d. The material has no microorganism nor
94. Multiple vial-dose-insulin when in use should be spores present that might cause an
a. Kept at room temperature infection
b. Kept in narcotic cabinet
c. kept in the refrigerator 100. In using liquid sterilizer versus autoclave
d. store in the freezer machine, which of the following is true?
a. Autoclave is better in sterilizing OR supplies
95. Insulins using insulin syringe are given using versus liquid sterilizer
how many degrees off needle insertion? b. They are both capable of sterilizing the
a. 45 equipments, however, it is necessary to
b. 180 soak supplies in the liquid sterilizer for a
c. 90 longer period of time
d. 15 c. Sharps are sterilizes using autoclave and not
cidex
Situation: Maintenance of sterility is an important d. If liquid sterilizer is used rinsing it before
function a nurse should perform in any OR setting. using is not necessary

96. Which of the following is true with regards to


sterility?
a. Sterility is time related, items are hot
considered sterile after a period of 30 days
of being not use.
b. For 9 months, sterile items are considered
sterile as long as they are covered with
sterile muslin cover and stored in a dust
proof covers.
c. Sterility is event related, not time related
d. For 3 weeks, items double covered with
muslin are considered sterile as long as they
have undergone the sterilization process
NURSING PRACTICE IV d. CT Scan and Incidence report

Situation: After an abdominal surgery, the circulating


and scrub nurses have critical responsibility about Situation: An entry level nurse should be able to apply
sponge and instrument count. theoretical knowledge in the performance of the basic
nursing skills.
1. Counting is performed thrice: during the
preincision phase, the operative phase and 6. A client has an indwelling urinary catheter and
closing phase. Who counts the sponges, needles she is suspected of having urinary infection.
and instruments? How should you collect a urine specimen for
a. The scrub nurse only culture and sensitivity?
b. The circulating nurse only a. Clamp tubing for 60 minutes and insert a
c. The surgeon and the assistant surgeon sterile needle into the tubing above the
d. The scrub nurse and the circulating nurse clamp to aspirate urine
b. Drain urine from the drainage bag into the
2. The layer of the abdomen is divided into 5. sterile container
Arrange the following from the first layer going c. Disconnect the tubing from the urinary
to the deepest layer: catheter and let urine flow into a sterile
1. Fascia container
2. Muscle d. Wipe the self-sealing aspiration port with
3. Peritoneum antiseptic solution and insert a sterile
4. Subcutaneously needle into the self-sealing port
5. Skin
a. 5,4,3,2,1 7. To obtain specimen for sputum culture and
b. 5,4,1,3,2 sensitivity, which of the following instruction is
c. 5,4,2,1,3 best?
d. 5,4,1,2,3 a. Upon waking up, cough deeply and
expectorate into container
3. When is the first sponge/instrument count b. Cough after pursed lip breathing
reported? c. Save sputum for two days in covered
a. Before closing the subcutaneous layer container
b. Before peritoneum is closed d. After respiratory treatment, expectorate
c. Before closing the skin into a container
d. Before the fascia is sutured
8. The best time for collecting the sputum
4. Like any nursing interventions, counts should be specimen for culture and sensitivity is:
documented. To whom does the scrub nurse a. Before retiring at night
report any discrepancy of counts so that b. Anytime of the day
immediate and appropriate action is instituted? c. Upon waking up in the morning
a. Anaesthesiologists d. Before meals
b. Surgeon
c. OR nurse supervisor 9. When suctioning the endotracheal tube, the
d. Circulating nurse nurse should:
a. Explain procedure to patient; insert
5. Which of the following are 2 interventions of catheter gently applying sunction.
the surgical team when an instrument was Withdrawn using twisting motion
confirmed missing? b. Insert catheter until resistance is met and
a. MRI and Incidence report then withdraw slightly, applying suction
b. CT Scan, MRI, incidence report intermittently as catheter is withdrawn
c. X-RAY and Incidence report c. Hyperoxygenate client insert catheter using
back forth motion
d. Insert suction catheter four inches into the
tube, suction 30 seconds using
twirling motion as catheter is withdrawn Nursing intervention includes:
a. Bed rest
10. The purpose of NGT IMMEDIATELY after an b. Warm moist soak
operation is: c. Early ambulation
a. For feeding or gavage d. Hot sitz bath
b. For gastric decompression
c. For lavage, or the cleansing of the Situation – Mang Felix, a 79 year old man who is brought
Stomach content to the Surgical Unit from PACU after a transurethral
d. For the rapid return of peristalsis resection. You are assigned to receive him. You noted
that he has a 3-way indwelling urinary catheter for
Situation – Mr. Santos, 50, is to undergo cystoscopy de continuous fast drip bladder irrigation which is
to multiple problems like scantly urination, hematuria connected to a straight drainage.
and dysuria.
16. Immediately after surgery, what would you
11. You are the nurse in charge in Mr. Santos. When expect his urine to be?
asked are the organs to be examined dring a. Light yello
cystoscopy, you will enumerate as follows: b. Bright red
a. Urethra, kidney, bladder, urethra c. Amber
b. Urethra, bladder wall, trigone, urethral d. Pinkish to red
opening
c. Bladder wall, uterine wall and urethral 17. The purpose of the continuous bladder
opening irrigation is to:
d. Urethral opening, ureteral opening a. Allow continuous monitoring of the fluid
bladder output status
b. Provide continuous flushing of clots and
12. In the OR, you will position Mr. Santos who is debris from the bladder
cystoscopy in: c. Allow for proper exchange of
a. Supine electrolytes and fluid
b. Lithotomy d. Ensure accurate monitoring of intake
c. Semi-fowler and output
d. Trendelenburg
18. Mang Felix informs you that he feels some
13. After cystoscopy, Mr. Santos asked you to discomfort on the hypogastric area and he has
explain why there is no incision of any kind. to void. What will be your most appropriate
What do you call him? action.
a. “Cystoscopy is direct visalization and a. Remove his catheter then allow him to
examination bu urologist”. avoid on his own
b. “Cystoscopy is done by x-ray b. Irrigate his catheter
visualization of the urinary tract”. c. Tell him to “Go ahead and void. You have
c. “Cystoscopy is done by using laser on the an indwelling catheter.”
urinary tract”. d. Assess color and rate of outflow, if
d. “Cystoscopy is an endoscopic procedure there is changes refer to urologist for
of the urinary tract”. possible irrigation.

14. Within 24-48 hours post cystoscopy, it is normal 19. You decided to check on Mang Felix’s IV fluid
to observe one of the following: infusion. You noted a change in flow rate, pallor
a. Pink-tinged urine and coldness around the insertion site. What is
b. Distended bladder your assessment finding?
c. Signs of infection a. Phlebitis
d. Prolonged hematuria b. Infiltration to subcutaneous tissue
c. Pyrogenic reaction
15. Leg cramps are NOT uncommon post d. Air embolism
cystoscopy.
20. Knowing that proper documentation of d. Fever, Irritability and a large output of
assessment findings and interventions are diluted urine
important responsibilities of the nurse during first
post-operative day, which of the following is the 25. What kind of renal failure will melamine
LEAST relevant to document in the case of Mang poisoning cause?
Felix? a. Chronic, Prerenal
a. Chest pain and vital signs b. Chronic, Intrarenal
b. Intravenous infusion rate c. Acute, Postrenal
c. Amount, color, and consistency of d. Accute, Prerenal
bladder irrigation dranaige
d. Activities of daily living started Situation: Leukemia is the most common type of
childhood cancer. Acute lymphoid leukemia is the cause
Situation: Melamine contamination in milk has brought of almost 1/3 of all cancer that occurs in children under
worldwide crisis both in the milk production sector as age 15.
well as the health and economy. Being aware of the
current events is one quality that a nurse should 26. The survival rate for Acute Lymphoid Leukemia
possess to prove that nursing is a dynamic profession is approximately:
that will adapt depending on the patient’s need. a. 25%
b. 40%
21. Melamine is a synthetic resin used for c. 75%
whiteboards, hard plastics and jewellery box d. 95%
covers due to its fire retardant properties. Milk
and food manufacturers add melamine in order 27. Whereas acute nonlymphoid leukemia has a
to: survival rate of:
a. It has a bacteriostatic property leading to a. 25%
increase food and milk life as a way of b. 40%
preserving foods c. 75%
b. Gives a glazy and more edible look on d. 95%
foods
c. Make milks more tasty and creamy 28. The three main consequence of leukemia that
d. Create an illusion of a high protein cause the most danger is:
content on their production a. Neutropenia causing infection, anemia
22. Most of the milk contaminated by melamine causing impaired oxygenation and
came from which country? thrombocytopenia leading to bleeding
a. India tendencies
b. China b. Central nervous system infiltration,
c. Philippines anemia causing impaired oxygenation
d. Korea and thrombocytopenia leading to
bleeding tendencies
23. Which government agency is responsible for c. Splenomegaly, hepatomegaly, fractures
testing the melamine content of foods and food d. Invasion by the leukemic cells to the
products? bone causing severe bone pain
a. DOH
b. MMDA 29. Gold standard in the diagnostic of leukemia is
c. NBI by which of the following
d. BFAD a. Blood culture and sensitivity
b. Bone marrow biopsy
24. Infants are the most vulnerable to melamine c. Blood biopsy
poisoning. Which of the following is NOT a sign d. CSF aspiration and examination
of melamine poisoning?
a. Irritability, back ache, Urolithiasis 30. Adriamycin, Vincristine, Prednisone, and L
b. High blood pressure, fever asparaginase are given to the client for long
c. Anuria, Oliguria or Hematuria term therapy. One common side effect
Especially of adriamcycin is alopecia. The child 34. Carmen, who is asking the nurse the most
ask: “Will I get my hair back once again?”The appropriate time of the month to do her self-
nurse best respond is by saying: examination of the breast. The MOST
a. “Don’t be silly, of course you will get appropriate reply by the nurse would be:
your hair back a. The 26th day of the menstrual cycle
b. “ We are not sue, let’s hope it’ll grow b. 7 to 8 days after conclusion of the
c. “This side effect is usually permanent. menstrual period
But I will get the doctor to discuss it for c. During the menstruation
you” d. The same day each month
d. “You hair will regrow in 3 to 6 months
but of different color, usually darker 35. Carmen being treated with radiation therapy.
and of different texture What should be included in the in the plan of
care to minimize skin damage from the
Situation: Breast cancer is the 2nd most common type of radiation therapy?
cancer after lung cancer and 99% of which, occurs in a. Cover the areas with thick clothing
woman. Survival rate is 98% if this is detected early and materials
treated promptly. Carmen is a 53 year old patient in the b. Apply a heating pad to the size
high risk group for break cancer was recently diagnosed c. Wash skin with water after the therapy
with Breast cancer. d. Avoid applying creams and powders to
31. All of the following are factors that said to the area
contribute to the development of breast cancer
except: 36. Based on the DOH and World Health
a. Prolonged exposure to estrogen such as Organization (WHO) guidelines, the mainstay
an early menarche or late menopause, for early detection method for breast cancer
nulliparity and childbirth after age 30 that is recommended for developing counties is:
b. Genetics a. A monthly breast self-examination (BSE)
c. Increasing age and an annual health worker breast
d. Prolonged intake of Tamoxifen examination (HWBE)
(Nolvadex) b. An annual hormone receptor assay
c. An annual mammogram
32. Protective factors for the development of d. A physician conduct a breast clinical
breast cancer includes which of the following examination every 2 years
except:
a. Exercise 37. The purpose of performing the breast self-
b. Breast feeding examination (BSE) regularly is to discover:
c. Prophylactic Tamoxifen a. Fibrocystic masses
d. Alcohol intake b. Area of thickness or fullness
33. A patient diagnosed with breast cancer has c. Cancerous lumps
been offered the treatment choices of breast d. Changes from previous BSE
conservation surgery with radiation or a
modified radical mastectomy. When questioned 38. If you are to instruct a postmenopausal woman
by the patient about these options, the nurse about BSE, when would you tell her to do BSE:
informs the patient that the lumpectomy with a. On the same day of each month
radiation: b. On the first day of her menstruation
a. Reduces the fear and anxiety that c. Right after the menstrual period
accompany the diagnosis and treatment d. On the last day of her menstruation
of cancer
b. Has about the same 10-year-survival rate 39. During breast self- examination, the purpose of
as the modified radical mastectomy standing in front of the mirror is to observe the
c. Provides a shorter treatment period with breast for:
a few long term complications a. Thickening of the tissue
d. Preserves the normal appearance and
Sensitivity of the breast.
b. Lumps in the breast tissue d. Ineffective tissue perfusion, peripheral,
c. Axillary lymphnodes celebral, cardiovascular,
d. Change in size and colour gastrointestinal, renal

40. When preparing to examine the left breast in a 45. What intervention should you include in your
reclining position, the purpose of placing a small care plan?
folded towel under the clients left shoulder is a. Inspect his skin for petechiae, bruising,
to: GI bleeding regularly
a. Bring the breast closer to the examiner’s b. Place albert on stick isolation precaution
right hand c. Provide rest in between activities
b. Tense the pectoral muscle d. Administer antipyretics if his
a. Balance the breast tissue more evenly temperature exceeds 38C
on the chest wall
b. Facilitate lateral positioning of the Situation: Burn are cause by transfer of heat source to
breast the body. It can be thermal, electrical, radiation, or
chemical
Situation- Radiation therapy is another modality of
cancer management. With emphasis on 46. A burn characterized by pale white appearance,
multidisciplinary management you have important charred or with fat exposed and painlessness is:
responsibilities as nurse. a. Superficial partial thickness burn
b. Deep partial thickness burn
41. Albert is receiving external radiation therapy c. Full thickness burn
and he complains of fatigue and malaise. Which d. Deep full thickness burn
of the following nursing interventions would be
most helpful to Albert? 47. Which of the following BEST describe superficial
a. Tell him that sometimes these feelings partial thickness burn or first degree burn?
can be psychogenic a. Structures beneath the skin are damage
b. Refer him to the physician b. Dermis is partially damaged
c. Reassure him that these feelings are c. Epidermis and dermis are both damaged
normal d. Epiderm is damaged
d. Help him plan his activities
48. A burn that said to be“WEEPING”is classified as:
42. Immediately following the radiation a. Superficial partial thickness burn
teletherapy, Albert is b. Deep partial thickness burn
a. Considered radioactive for 24 hours c. Full thickness burn
b. Given a complete bath d. Deep full thickness burn
c. Placed on isolation for 6 hours
d. Free from radiation 49. During the acute phase of the burn injury,
which of the following is a priority?
43. Albert is admitted with a radiation induced a. Wound healing
thrombocytopenia. As a nurse you should b. Emotional support
observe the following symptoms: c. Reconstructive surgery
a. Petechiae, ecchymosis, epistaxis d. Fluid resuscitation
b. Weakness, easy fatigability, pallor
c. Headache, dizziness, blurred vision 50. While in the emergent phase, the nurse knows
d. Severe sore throat, bacteremia, that the priority is to:
hepatomegaly a. Prevent infection
44. What nursing diagnosis should be of highest b. Prevent deformities and contractures
priority? c. Control pain
a. Knowledge deficit regarding d. return the hemodynamic stability fluid
thrombocytopenia precautions resuscitation
b. Activity intolerance
c. Impaired tissue integrity
51. The MOST effective method of delivering pain The client is developing:
medication during the emergent phase is: a. Celebral hypoxia
a. intramuscularly b. Metabolic acidosis
b. orally c. Hypervolemia
c. subcutaneously d. Renal failure
d. intravenously
58. 1 165 lbs trauma client was rushed to the
52. When a client accidentally splashes chemicals to emergency room with full thickness burns on
his eyes, the initial priority care following the whole face, right and left arm, and at the
chemical burn is to: anterior upper chest sparing the abdominal
a. irrigate with normal saline for 1-15 area. He also has superficial partial thickness
minutes burn at the posterior trunk and at the half
b. transport to a physician immediately upper portion of the left leg. He is at the
c. irrigate with water for 15 minutes or emergency phase of burn. Using the parkland’s
longer formula, you know that during the first 8 hours
d. cover the eyes with sterile gauze of burn, the amount of fluid will be given is:
53. Which of the following can be a fatal a. 5,400 ml
complication of upper airway burns? b. 9,450 ml
a. stress ulcers c. 10,800 ml
b. shock d. 6,750 ml
c. hemorrhage
d. laryngeal spasms and swelling 59. The doctor incorporated insulin on the client’s
fluid during the emergency phase. The nurse
54. When a client rush towards you and he has a knows that insulin is given because:
burning clothes on, it is your priority to do a. Client with burn also develops metabolic
which of the following first? acidosis
a. log roll on the grass/ground b. Client with burn also develops
b. slap the flames with his hands hyperglycemia
c. try to remove the burning clothes c. Insulin is needed for additional energy
d. splash the client with 1 bucket of cool and glucose burning after the stressful
water incidence to hasten wound healing,
regain of consciousness and rapid return
55. Once the flames are extinguished, it is most of hemodynamic stability
important to: d. For hyperkalemia
a. cover client with warm blanket
b. give him sips of water 60. The IV fluid of choice for burn as well as
c. calculate the extent of his burns dehydration is:
d. assess the Sergio’s breathing a. 0.45% NaCI
b. Sterile water
56. During the first 24 hours after the thermal c. NSS
injury, you should assess Sergio for: d. D5LR
a. hypokalemia and hypernatremia
b. hypokalemia and hyponatremia Situation: ENTEROSTOMAL THERAPY is now considered
c. hyperkalemia and hyponatremia specialty in nursing. You are participating in the
d. hyperkalemia and hypernatremia OSTOMY CARE CLASS.

57. A client was sustained deep partial thickness 61. You plan to teach Fermin how to irrigate the
and full thickness burns of the face, whole colostomy when:
anterior chest and both upper extremities two a. The perineal wound heals and Fermin
days ago begins to exhibit extreme restlessness. can sit comfortably on the commode
You recognize that this most likely indicates that b. ‘fermin can lie on the side comfortably,
about the 3rd postoperative day
c. The abdominal incision is closed and
Contamination is no longer a danger It is important for nurses to gather as much information
d. The stools starts to become formed, to be able to address their needs for nursing care.
around the 7th postoperative day
66. Critically ill patients frequently complain about
62. When preparing to teach Fermin how to irrigate which of the following when hospitalized?
colostomy, you should plan to do the a. Hospital food
procedure: b. Lack of privacy
a. When Fermin would have normal bowel c. Lack of blankets
movement d. Inadequate nursing staff
b. At least 2 hours before visiting hours
c. Prior to breakfast and monitoring care 67. Who of the following is at greatest risk of
d. After Fermin accepts alteration in body developing sensory problem?
image a. Female patient
b. Transplant patient
63. When observing a return demonstration of a c. Adolescent
colostomy irrigation, you know that more d. Unresponsive patient
teaching is required if Fermin:
a. Lubricates the tip of the catheter prior to 68. Which of the following factors may inhibit
inserting into the stoma learning in critically ill patients?
b. Hangs the irrigating bag on the a. Gender
bathroom door cloth hook during fluid b. Educational level
insertion c. Medication
c. Discontinues the insertion of fluid after d. Previous knowledge of illness
500 ml of fluid has been instilled
d. Clamps of the flow of fluid when felling 69. Which of bthe following statements does not
uncomfortable. apply to critically ill patients?
a. Majority need extensive rehabilitation
64. You are aware that teaching about colostomy b. All have been hospitalized previously
care is understood when Fermin states “I will c. Are physically unstable
contact my physician and report: d. Most have chronic illness
a. If I have any difficulty inserting the
irrigating tub into the stoma.” 70. Families of critically ill patients desire which of
b. If I noticed a loss sensation to touch in the following needs to be met first by the
the stoma tissue.” nurse?
c. The expulsion of flatus while the a. Provision of comfortable space
rrigating fluid is running out.” b. Emotional support
d. When mucus is passed from the stoma c. Updated information on clients status
between the irrigations.” d. Spiritual counselling

65. You would know after teaching Fermin that Situation: Johnny, sought consultation to the hospital
dietary instruction for him is effective when he because of fatigability, irritability, iittery and he has
states “It is important that I eat: been experiencing this sign and symptoms for the past 5
a. Soft food that is easily digested and months.
absorbed by my large intestines.” 71. His diagnosis was hyperthyroidism, the
b. Bland food so that my intestines do not following are expected symptoms except:
become irritated.” a. Anorexia
c. Food low in fiber so that there are fewer b. Fine tremors of the hnad
stools.” c. Palpitation
d. Everything that I ate before the d. Hyper alertness
operation while avoiding foods that
cause gas.” 72. She has to take drugs to treat her
Situation: Based on studies of nurses working in special hyperthyroidism. Which of the following will
units like intensive care unit and coronary care unit you NOT expect that the doctor will prescribe?
a. Colace (Docusate) To:
b. Tapazole (Methimazole) a. Decrease the vascularity and size of the
c. Cytomel (Liothyronine) thyroid gland
d. Synthroid(Levothyroxine) b. Decrease the size of the thyroid gland
only
73. The nurse knows that Tapazole ha which of the c. Increase the vascularity and size of the
following side effect that will warrant thyroid gland
immediate withholding of the medication? d. Increase the size of the thyroid gland
a. Death only
b. Hyperthermia
c. Sore throat 79. Which of the following is a side effect of lugol’s
d. Thrombocytosis solution?
a. Hypokalemia
74. You asked questions as soon as she regained b. Enlargement of the thyroid gland
consciousness from thyroidectomy primarily to c. Nystagmus
assess the evidence of : d. Excessive salivation
a. Thyroid storm
b. Damage to the laryngeal nerve 80. In administering Lugol’s solution, the
c. Mediastinal shift precautionary measure should include:
d. Hypocalcaemia tetany a. Administer with glass only
e. b. Dilute with juice and administer with a
straw
75. Should you check for haemorrhage, you will: c. Administer it with milk and dink it
a. Slip your hand under the nape of her d. Follow it with milk magnesia
neck
b. Check for hypotension Situation: Pharmacological treatment was not effective
c. Apply neck collar to prevent for Johnny’s hyperthyroidism and now, he is scheduled
haemorrhage for thyroidectomy.
d. Observe the dressing if it is soaked with
blood 81. Instrument in the surgical suite for surgery is
classified as either CRITICAL, SEMI CRITICAL, and
76. Basal metabolic rate is assessed on Johnny to NON CRITICAL. If the instruments are
determine his metabolic rate. In assessing the introduced directly into the blood stream or
BMR using the standard procedure, you need to into any normally sterile cavity or area of the
tell Johny that: body it is classified as:
a. Obstructing his vision a. Critical
b. Restraining his upper and lower b. Non Critical
extremities c. Semi Critical
c. Obstructing his hearing d. Ultra critical
d. Obstructing his nostrils with a clamp
82. Instruments that do not touch the patient or
77. The BMR is based on the measurement that: have contact only to intact skin is classified as:
a. Rate of respiration under different a. Critical
condition of activities and rest b. Non critical
b. Amount of oxygen consumption under c. Semi critical
resting condition over a measured d. Ultra critical
period of time
c. Amount of oxygen consumption under 83. If an instrument is classified as Semi Critical, an
stressed condition over a measured acceptable method of making the instrument
period of time ready for surgery is through:
d. Ratio of respiration to pulse rate over a a. Sterilization
measured period of time b. Disinfection
78. Her Physician ordered lugol’s solution in order c. Decontamination
d. Cleaning d. Tetany
84. While critical items and should be: 90. After surgery Johnny develops peripheral
a. Clean numbness, tingling and muscle twitching and
b. Sterilized spasm. What would you anticipate to
c. Decontaminated administer?
d. Disinfected a. Magnesium sulphate
b. Potassium iodide
85. As a nurse, you know that intact skin acts as an c. Calcium gluconate
effective barrier to most microorganisms. d. Potassium chloride
Therefore, items that come in contact with the
intact skin or mucus membranes should be: Situation: Budgeting is an important part of a nurse
a. Disinfected Managerial activity. The correct allocation and
b. Clean distribution of resources is vital in the harmonious
c. Sterile operation of the financial balance of the agency.
d. Alcoholized
91. Which of the following best defines Budget?
86. You are caring for Johnny who is scheduled to a. Plan for the allocation of resources for
undergo total thyroidectomy because of a future use
diagnosis of thyroid cancer, prior to total b. The process of allocating resources for
thyroidectomy, you should instruct Johnny to: future use
a. Perform range and motion exercise on c. Estimate cost of expenses
the head and neck d. Continous process in seeing that the
b. Apply gentle pressure against the goals and objective of the agency is met
incision when swallowing
c. Cough and deep breathe every 2 hours 92. Which of the following is best defines Capital
d. Support head with the hands when Budget?
changing position a. Budget to estimate the cost of direct
labour, number of staff to be hired and
87. As Johnny’s nurse, you plan to set up necessary number of workers to meet
emergency equipment at her bedside following the general patient needs
thyroidectomy, You should include: b. Includes the monthly and daily expenses
a. An airway and rebreathing tube and expected revenue and expenses
b. A trancheostomy set and oxygen c. Theses are related to long term
c. A crush cart with bed board planning and includes major
d. Two ampoules of sodium bicarbonate replacement or expansion of the plant,
major equipment and inventories
88. Which of the following nursing interventions is d. These are expenses that are not
appropriate after a total thyroidectomy? dependent on the level of production or
a. Place pillows under your patient’s sales, They tend to be time-related, such
shoulder as salaries nor rents being paid per
b. Raise the knee-gatch to 30 degrees month
c. Keep you patient in a high-fowler’s
position 93. Which of the following best described
d. Support the patient’s head and neck Operational Budget?
with pillows and sandbags a. Budget to estimate the cost of direct
labour, number of staff to be hired and
89. If there is an accidental injury to the necessary number of workers to meet
parathyroid gland during a thyroidectomy which the general patient needs
of the following might Leda develops b. Includes the monthly and daily
postoperatively? expenses and expected revenue and
a. Cardiac arrest expenses
b. Respiratory failure c. these are related to long term planning
c. Dyspnea and includes major replacement or
expansion of the plant, major equipments
and inventories. 98. Which of the following is true with the Triad
d. These are expenses that are not seen in head injuries?
dependent on the level of production or a. Narrowing of Pulse pressure, Cheyne
sales. They tend to be time-related,such as stokes respiration, Tachycardia
rent b. Widening Pulse pressure, Irregular
respiration, Bradycardia
94. Which of the following accurately describes a c. Hypertension, Kussmaul’s respiration,
Fixed Cost in budgeting?
Tachycardia
a. There are usually the raw materials and
d. Hypotension, Irregular respiration,
labour salaries that depend on the
Bradycardia
production or sales
b. These are expenses that change in
proportion to the activity of a business 99. In a client with a cheyne stokes respiration,
c. These are expenses that are not Which of the following is the most appropriate
dependent on the level of production or nursing diagnosis
sales. They tend to be time-related, such a. Ineffective airway clearance
as rent b. Impaired gas exchange
d. This is the summation of the Variable Cost c. Ineffective breathing pattern
and the Fixed Cost d. Activity Intolerance

95. Which of the following accurately describes 100. You know the apnea is seen in client’s
Variable Cost in budgeting? with cheyne stokes respiration, APNEA is
a. These are related to long term planning defined as:
and include major replacement or a. Inability to breathe in a supine position
expansion of the plant, major equipments so the patient sits up in the bed to
and inventories breathe
b. These are expenses that changes in b. The patient is dead, the breathing stops
proportion to the activity of a business
c. There is an absence of breathing for a
c. These are expenses that are not
period of time, usually 15 seconds or
dependent on the level of production or
more
sales. They tend to be time-related, such
d. A period of hypercapnea and hypoxia
as rent
d. This is the summation of the Variable Cost due to the cessation of respiratory effort
and the Fixed Cost inspite of normal respiratory functioning

Situation – Andrea is admitted to the ER following an


assault where she was hit in the face and head. She was
brought to the ER by a police woman. Emergency
measures were started.

96. Andrea’s respiration is described as waxing and


waning. You know that this rhythm of respiration
is defined as waxing and waning. You know that
this rhythm of respiration is defined as:
a. Biot’s
b. Cheyne Strokes
c. Kussmaul’s
d. Eupnea

97. What do you call the triad of sign and symptoms


seen in a client with increasing ICP?
a. Virchow’s Triad
b. Nursing triad
NURSING PRACTICE V
c. The Chinese Triad
d. Charcot’s Triad
Situation: Understanding different models of acre is a 5. A client state, “I get down on myself when I
necessary part of the nurse patient relationship. make mistake.” Using cognitive therapy
approach, the nurse should:
1. The focus of this therapy is to have a positive a. Teach the client relaxation exercise to
environmental manipulation, physical and social diminish stress
to effect a positive change. b. Provide the client with Mastery experience
a. Milleu to boost self esteem
b. Psychotherapy c. Explore the client’s past experience that
c. Behavior causes the illness
d. Group d. Help client modify the belief that anything
less than perfect is horrible
2. The client asks the nurse about milleu therapy.
The nurse responds knowing that the primary 6. The most advantageous therapy for a preschool
focus of milleu therapy can be best described by age child with a history of physical and sexual
which of the following? abuse would be:
a. A form of behavior modification therapy a. Play
b. A cognitive approach of changing the b. Psychoanalysis
behaviour c. Group
c. A living, learning or working environment d. Family
d. A behavioural approach to changing
behaviour 7. An 18 year old client is admitted with the
diagnosis of anorexia nervosa. A cognitive
3. A nurse is caring for a client with phobia who is behavioural approach is used as part of her
being treated for thw condition. The client is treatment plan. The nurse understand that the
introduced to short periods of exposure to the purpose of this approach is to:
phobic object while in relaxed state. The nurse a. Help the client identify and examine
understands that this form of behavior dysfunctional thoughts and beliefs
modification can be best described as b. Emphasize social interaction with clients
a. Systematic desensitization who withdraw
b. Self-control therapy c. Provide a supportive environment and a
c. Aversion Therapy therapeutic community
d. Operant conditioning d. Examine Intrapsychic conflict and past
events in life
4. A client with major depression is considering
cognitive therapy. The client say to the nurse,
“How does this treatment works?” the nurse 8. The nurse is preparing to provide reminiscence
responds by telling the client that: therapy for a group of client. Which of the
a. N following clients will be nurse select for this
b. “This type of treatment helps you examine group?
how your past life has contributed to your a. A client who experiences profound
problems.” depression with moderate cognitive
c. “This type of treatment helps you to impairment
confront your fears by exposing you to the b. a catatonic, Immobile client with moderate
feared object abruptly” cognitive impairment
d. “This type of treatment will help you relax c. An undifferentiated schizophrenic client
and develop new coping skills.” with moderate cognitive impairment
d. A client with mild depression who exhibits
who demonstrates normal cognition

9. Which intervention would be typical of a nurse


using cognitive-behavioral approach to a client
experiencing low self-esteem?
a. Use of unconditional positive regard c. refer the client to the psychiatrist
b. Analysis of free association d. refer the matter to the police
c. Classical conditioning
d. Examination of negative thought patterns Situation: Rose seeks psychiatric consultation because
of intense fear of flying in an airplane which has gently
10. Which of the following therapies has been affected her chances of successful in her job.
strongly advocated for the treatment of post-
traumatic stress disorders? 16. The most common defense mechanism used by
a. ECT phobic client is:
b. Group Therapy a. Supression
c. Hypnotherapy b. Denial
d. Psychoanalysis c. Rationalization
d. Displacement
11. The nurse knows that im group therapy, the
maximum number of members to include is: 17. The goal of the therapy in phobia is:
a. 4 a. Change her lifestyle
b. 8 b. Ignore tension producing situation
c. 10 c. Change her reaction towards anxiety
d. 16 d. Eliminates fear producing situations

12. The nurse is providing information to a client 18. The therapy most effective for client’s with
with the use of disulfiram (antabuse) for the phobia is:
treatment of alcohol abuse. The nurse a. Hypnotherapy
understands that this form of therapy works on b. Cognitive therapy
what principle? c. Group therapy
a. Negative Reinforcement d. Behavior therapy
b. Operant conditioning
c. Aversion therapy 19. The fear and anxiety related to phobia is said to
d. Gestait therapy be abruptly decreased when the patient is
exposed to what is feared through:
13. A biological or medical approach in treating a. Guided Imagery
psychiatric patient is: b. Systematic desensitization
a. Million therapy c. Flooding
b. Behavioral therapy d. Hypotherapy
c. Somatic therapy
d. Psychotherapy 20. Based on the presence of symptom, the
appropriate nursing diagnosis is:
14. Which of these nursing actions belong to the a. Self-esteem disturbance
secondary level of preventive intervention? b. Activity intolerance
a. Providing mental health consultation to c. Impaired adjustment
health care providers d. Ineffective individual coping
b. Providing emergency psychiatric services
c. Being politically active in relation to mental Situation: Mang Jose, 39 year old farmer, unmarried had
health issues been confined in the National center for mental health
d. Providing mental health education to for three years with a diagnosis of Schizophrenia.
members of the community
21. The most common defense mechanism used by
15. When the nurse identifies a client who has a paranoid client is:
attempted to commit suicide the nurse should: a. Displacement
a. call a priest b. Rationalization
b. counsel the client c. Suppression
d. Projection
22. Nursing…..
a. Kdknf
b. Nfdjnvgj Is best described in one of the following statements:
c. Bvxnbv a. Unacceptable feelings or behavior are kept out
d. Nbnbvf of awareness by developing the opposite
behavior or emotion
23. A relevant nursing diagnosis for clients with b. Consciously unacceptable instinctual drives are
auditory hallucination is: diverted into personally and socially acceptable
a. Sensory perceptual alteration channels
b. Altered thought process c. Something unacceptable already done
c. Impaired social interaction d. Transfer of emotions associated with a
d. Impaired verbal communication particular person, object or situation to another
less threatening person, object or situation
24. During mealtime, Jose refused to eat telling that
the food was poisoned. The nurse should: 29. To be more effective, the nurse who cares for
a. gnore his remark persons with obsessive compulsive disorder
b. Offer him food in his own container must posses one of the following qualities:
c. Show him how irrational his thinking is a. Compassion
d. Respect his refusal to eat b. Patience
c. Consistency
25. When communicating with Jose, The nurse d. Friendliness
considers the following except:
30. Persons with OCD usually manifest:
a. Be warm and enthusiastic
a. Fear
b. Refrain from touching Jose
b. Apathy
c. Do not argue regarding his hallucination c. Suspiciousness
and delusion d. Anxiety
d. Use simple, clear language
e. Situation: The patient who is depressed will undergo
Situation: Gringo seeks psychiatric counseling for his electroconvulsive therapy:
ritualistic behaviour of counting his money as many as
10 times before leaving home. 31. Studies on biological depression support
electroconvulsive therapy as mode of treatment.
26. An initial appropriate are nursing diagnosis is: That rationale is:
a. Impaired social interaction a. ECT produces massive brain damage which
b. Ineffective individual coping destroys the specific area containing
c. Impaired adjustment memories related to the events surrounding
d. Anxiety moderate the development of psychotic condition
b. The treatment serve as a symbolic
27. Obsessive compulsive disorder is BEST described punishment for the client who feels guilty
by: and worthless
a. Uncontrollable impulse to perform an act or c. ECT relieves depression psychologically by
ritual repeatedly increasing the norepinephrine level
b. Persistent thoughts d. ECT is seen as a life-threatening experience
c. Recurring unwanted and disturbing though and depressed patients mobilized all their
alternation with a behavior bodily defences to deal with this attack
d. Pathological persistence of unwilled
thought, feeling or impulse 32. The preparation of a patient for ECT ideally is
MOST similar to preparation for a patient for:
28. The defense mechanism used by persons with a. Electroencephalogram
obsessive compulsive disorder is undoing and it
b. General anaesthesia a. Emotional crisis
c. X-ray b. Cholinergic crisis
d. Electrocardiogram c. Menopausal crisis
d. Myasthenia crisis
33. Which of the following is a possible side effect
which you will discuss with the patient? 40. If you are not extra careful and by chance you
a. Haemorrhage with the brain give over medication, this would lead to:
b. Encephalitis a. Cholinergic crisis
c. Robot-like body stiffness b. Menopausal crisis
d. Confusion, disorientation and short term c. Emotional crisis
memory loss d. Myasthenia crisis

34. Informed consent is necessary for the treatment Situation: Rosanna 20 y/o unmarried patient believes
for involuntary clients. When this cannot be that the toilet for the female patient in contaminated
obtained, permission may be taken from the: with AIDS virus and refuses to use it unless she flushes it
a. social worker three times and wipes the seat same number of times
b. next of kin or guardian with antiseptic solution.
c. doctor
d. chief nurse 41. ??

35. after ECT, the nurse should do this action before a. Adjust to a strange environment
giving the client fluids, food or medication: b. Express her anxiety
a. assess the grag reflex c. Develop the sense of trust in other person
b. next of kin or guardian d. Control unacceptable impulses or feelings
c. assess the sensorium
d. check 02 sat with a pulse oximeter 42. Assessment data upon admission help the nurse
to identify this appropriate nursing diagnosis
Situation: Mrs Ethel Agustin 50 y/o, teacher is afflicted a. Ineffective denial
with myasthenia gravis. b. Impaired adjustment
c. Ineffective individual coping
36. Looking at Mrs Agustin, your assessment would d. Impaired social interaction
include the following except:
a. Nystagmus 43. An effective nursing intervention to help Rosana
b. Difficulty of hearing is;
c. Weakness of the levator palpebrae a. Convincing her to use the toilet after the
d. Weakness of the ocular muscle nurse has used it first
b. Explaining to her that AIDS cannot be
37. In an effort to combat complications which transmitted by using the toilet
might occur relatives should he taught; c. Allowing her to flush and clear the toilet
a. Checking cardiac rate seat until she can manage her anxiety
b. Taking blood pressure reading d. Explaining to her how AIDS is transmitted
c. Techniques of oxygen inhalation
d. Administration of oxygen inhalation 44. The goal for treatment for Rosana must be
directed toward helping her to;
38. The drug of choice for her condition is; a. Walk freely about her past experience
a. Prostigmine b. Develop trusting relationship with others
b. Morphine c. Gain insight that her behaviour is due to
c. Codeine feeling of anxiety
d. Prednisone d. Accept the environment unconditionally

39. As her nurse, you have to be cautious about


administration of medicine, if she is under
medicated this can cause;
45. Psychotherapy which is prescribed for Rosana is b. Self-esteem disturbance
described as; c. Ineffective individual coping
a. Establishing an environment adapted to an d. Defensive coping
individual patient needs
b. Sustained interaction between the therapist 50. Most appropriate nursing intervention for a
and client to help her develop more client with suspicious behaviour is one of the
functional behaviour following;
c. Using dramatic techniques to portray a. Talk to the client constantly to reinforce
interpersonal conflicts reality
d. Biologic treatment for mental disorder b. Involve him in competitive activities
c. Use Non Judgemental and Consistent
Situation: Dennis 40 y/o married man, an electrical approach
engineer was admitted with the diagnosis of paranoid d. Project cheerfulness in interacting with the
disorder. He has become suspicious and distrustful 2 patient
months before admission. Upon admission, he kept on
saying, “my wife has been planning to kill me.” Situation: clients with Bipolar disorder receives a very
high nursing attention due to the increasing rate of
46. A paranoid individual who cannot accept the suicide related to the illness.
guilt demonstrate one of the following defense
mechanism; 51. The nurse is assigned to care for a recently
a. Denial admitted client who has attempted suicide.
b. Projection What should the nurse do?
c. Rationalization a. Search the client’s belongings and room
d. Displacement carefully for items that could be used to
attempted suicide.
47. One morning, Dennis was seen tilting his head as b. Express trust that the client won’t cause
if he was listening to someone. An appropriate self-harm while in the facility.
nursing intervention would be; c. Respect the client’s privacy by not
a. Tell him to socialize with other patient to searching any belongings.
divert his attention d. Remind all staff members to check on
b. Involve him in group activities the client frequently.
c. Address him by name to ask if he is hearing
voice again 52. In planning activities for the depressed client,
d. Request for an order of antipsychotic especially during the early stages of
medicine hospitalization, which of the following plan is
48. When he says, “These voices are telling me my best?
wife is going to kill me.” A therapeutic a. Provide an activity that is quiet and solitary
communication of the nurse is which one of the to avoid increased fatigue such as working
following; on a puzzle and reading a book.
a. “i do not hear the voices you say you hear” b. Plan nothing until the client asks to
b. “are you really sure you heard those participate in the milieu
voices?” c. Offer the client a menu of daily activities and
c. “I do not think you heard those voices?” ask the client to participate in all of them
d. “whose voices are those?” d. Provide a structured daily program of
activities and encourage the client to
49. The nurse confirms that Dennis is manifesting participate
auditory hallucination. The appropriate nursing 53. A client with a diagnosis of major depression,
diagnosis she identifiesis; recurrent with psychotic features is admitted to
a. Sensory perceptual alteration the mental health unit. To create a safe
environment for the client, the nurse most
Importantly devises a plan of care that deals The nurse would initially:
specifically with the clients: a. Ask the client to leave the group session
a. Disturbed thought process b. Tell the client that she will not be
b. Imbalanced nutrition allowed to attend any more group
c. Self-care deficit sessions.
d. Deficient knowledge c. Tell the client that she needs to allow
other client in a group time to talk
54. The client is taking a tricyclic anti-depressant, d. Ask another nurses to escort the client
which of the following is an example of TCA? out of the group session.
a. Paxil
b. Nardil 59. A professional artist is admitted to the
c. Zoloft psychiatric unit for treatment of bipolar disorder.
d. Pamelor During last 2 weeks, the client has created 154
paintings, slept only 2 to 3 hours every 2 days,
55. The client visits the physician’s office to seek and lost 18 lb (8.2 kg). Based on Maslow’s
treatment for depression, feelings of hierarchy of needs, what should the nurse
hopelessness, poor appetite, insomnia, fatigue, provide this client with first?
low self-esteem, poor concentration, and a. The opportunity to explore family dynamics
difficulty in making decisions. The client states b. Help with re-establishing a normal sleep
that these symptoms began at least 2 years ago. pattern
Base on this report, the nurse suspects: c. Experiences that build self-esteem
a. Cyclothymic disorder d. Art materials and equipment
b. Bipolar disorder
c. Major depression 60. The physician orders lithium carbonate
d. Dysthymic disorder (Lithonate) for a client who’s in the manic phase
of bipolar disorder. During lithium therapy, the
56. The nurse is planning activities for a client who nurse should watch for which adverse reactions?
has bipolar disorder, which aggressive social a. Anxiety, restlessness, and sleep disturbance
behaviour. Which of the following activities b. Nausea, diarrhea, tremor, and lethargy
would be most appropriate for this client? c. Constipation, lethargy, and ataxia
a. Ping pong d. Weakness, tremor, and urine retention
b. Linen delivery
c. Chess Situation – Annie has a morbid fear of heights. She asks
d. Basketball the nurse what desensitization therapy is:

57. The nurse assesses a client with admitted 61. The accurate information of the nurse of the
diagnosis of bipolar affective disorder, mania. goal of desensitization is:
The symptoms presented by the client that a. To help the client relax and progressively
requires the nurse’s immediate intervention is work up a list of anxiety provoking
the client’s: situations through imagery
a. Outlandish behaviour and inappropriate b. To provide corrective emotional experiences
dress through a one-to-one intensive relationship
b. Grandiose delusion of being a royal c. To help clients in a group therapy setting to
descendant of king arthut take on specific roles and re-enact in front of
c. Nonstop physical activity and poor an audience, situation in which interpersonal
nutritional intake conflict is involved
d. Constant incessant talking that includes d. To help clients cope with their problems by
sexual topics and teasing the staff learning behaviors that are more functional
and be better equipped to face reality and
58. A nurse is conducting a group therapy session make decisions.
and during the session, A client with mania
consistently talks and demonstrate the group.
The behaviour is disrupting the group interaction.
62. It is essential in a desensitization for the patient Problem. After the identification of the research
to: problem, which of the following should be done?
a. Have rapport with the therapist a. Methodology
b. Use deeps breathing or another relaxation b. Acknowledgment
technique c. Review of related literature
c. Assess one’s self for the need of an anxiolytic d. Formulate hypothesis
drug
d. Work through unresolved unconscious 68. Which of the following communicate the results
conflicts of the research to the readers. They facilitate the
description of the data.
63. In this level of anxiety, cognitive capacity a. Hypothesis
diminishes. Focus becomes limited and client b. Research problem
experiences tunnel vision. Physical signs of c. Statistics
anxiety become more pronounced. d. Tables and graphs
a. Severe anxiety
b. Mild anxiety 69. In quantitative date, which of the following
c. Panic described as the distance in the scoring unites of
d. Moderate anxiety the variable from highest to the lower?
A. Frequency
64. Antianxiety medications should be used with B. Median
extreme caution because long term use can lead C. Mean
to: D. Range
a. Parkinson like syndrome
b. Hepatic failure 70. The expresses the variability of the data in
c. Hypertensive crisis reference to the mean. It provides as with a
d. Risk of addiction numerical estimate of how far, on the average
the separate observation are from the mean:
65. The nursing management of anxiety related with a. Mode
post- traumatic stress disorder includes all of the b. Median
following EXCEPT: c. Standard deviation
a. Encourage participation in recreation of d. Frequency
sports activities
b. Reassure client’s safety while touching Situation: Survey and Statistics are important part of
client research that is necessary to explain the characteristics
c. Speak in a calm soothing voice of the population.
d. Remain with the client while fear level is
high 71. According to the WHO statistics on the homeless
population around the world, which of the
SITUATION: you are fortunate to be chosen as part of following groups of people in the world
the research team in the hospital. A review of the disproportionately represents the homeless
following IMPORTANT nursing concepts was made. population
a. Hispanics
66. As a professional, a nurse can do research for b. Asians
varied reason except: c. African Americans
a. Professional advancement through research d. Caucasians
participation
b. To validate results of new nursing modalities 72. All but one of the following is not a measure of
c. For financial gains Central Tendency:
d. To improve nursing care a. Mode
b. Standard deviation
67. Each nurse participants was asked to identify a c. Variance
d. Range
73. In the value: 87, 85, 88, 92, 90; what is the a. There is a control group
mean? b. There is an experimental group
a. 88.2 c. Selection of subjects in the control group is
b. 88.4 randomized
c. 87 d. There is a careful selection of subjects in the
d. 90 experimental group

74. In the value: 80, 80, 80, 82, 82, 90, 90, 100; 80. The researcher implemented a medication
What is the mode? regimen using a new type of combination drug
a. 80 to manic patients while another group of manic
b. 82 patient receives the routine drugs. The
c. 90 researcher however handpicked the
d. 85.5 experimental group for they are the clients with
multiple episodes of bipolar disorder. The
75. In the value: 80, 80, 10 10, 25, 65, 100, 200; researcher utilized which research design?
What is the median? a. Quasi- experimental
A. 71.25 b. Phenomenological
B. 22.5 c. Pure experimental
C. 10 and 25 d. Longitudinal
D. 72.5
Situation 19: as a nurse, you are expected to participate
76. Draw lots, lottery table of random numbers or a in initiating or participating in the conduct of research
sampling that ensures that each element of the studies to improve nursing practice. You to be updated
population has an equal and independent on the latest trends and issues affected the professional
chance of being chosen is called: and the best practices arrived at by the profession.
a. Cluster
b. Stratified 81. You are interested to study the effects of
c. Simple mediation and relaxation on the pain
d. Systematic experienced by cancer patients. What type of
variable is pain?
77. An investigator wants to determine some of the a. Dependent
problems that are experienced by diabetic b. Independent
clients when using an insulin pump. The c. Correlational
investigator went into a clinic where the d. Demographic
personally knows several diabetic clients having
problem with insulin pump. The type of sampling 82. You would like to compare the support system of
done by the investigator is called: patient with chronic illness to those with acute
a. Probability illness. How will you best state your problem?
b. Snowball a. A descriptive study to compare the support
c. Purposive system of patients with chronic illness and
d. Incidental those with acute illness in terms of
demographic data and knowledge about
78. If the researcher implemented a new structured intervention
counselling program with a randomized group of b. The effects of the types of support system
subject and a routine counselling with another of patient with chronic illness and those
randomized group of subject, the research is with acute illness
utilizing which design? c. A comparative analysis of the support
a. Quasi Experimental system of patients with chronic illness and
b. Comparative those with acute illness
c. Experimental d. A study to compare the support system of
d. Methodological
patients with chronic illness and those with
acute illness.
79. Which of the following is not true about a pure
experimental research?
e. What are the differences of the support 87. Which of the following studies is based on
system being received by patient with quantitative research?
chronic illness and patients with acute a. A study examining the bereavement process
illness? in spouse of clients with terminal cancer
b. A study exploring the factors influencing
83. You would like to compare the support system weight control behaviour
o0f patients with chronic illness to those with c. A study measuring the effects of sleep
acute illness. Considering that the hypothesis deprivation on wound healing
was: “Clients with chronic illness have lesser d. A study examining client’s feelings before,
support system than clients with acute illness.” during and after bone marrow aspiration.
What type of research is this?
a. Descriptive 88. Which of the following studies is based on the
b. Correlational, Non Experimental qualitative research?
c. Experimental a. A study examining clients reaction to stress
d. Quasi Experiemental after open heart surgery
b. A study ,measuring nutrition and weight
84. In any research study where individual persons loss/gain in clients with cancer
are involved, it is important that an informed c. A study examining oxygen levels after
consent of the study is obtained. The following endotracheal suctioning
are essential information about the consent that d. A study measuring difference in blood
you should disclose to the prospective subjects pressure before, during and after procedure
except:
a. Consent to incomplete disclosure 89. An 85 year old client in a nursing home tells a
b. Description of benefits, risks, and nurse “ I signed the papers of that research study
discomforts because the doctor is so insistent and I want him
c. Explanation of procedure to taking care of me.” Which client right is being
d. Assurance of anonymity and confidentiality violated?
a. Right of self determination
85. In the Hypothesis: “The utilization of technology b. Right to full disclosure
in teaching improves the retention and c. Right to privacy and confidentiality
attention of the nursing students. “Which is the d. Right not to be harmed
dependent variable?
a. Utilization of technology 90. A supposition or system od idea that is proposed
b. Improvement in the retention and to explain a given phenomenon best defines:
attention a. A paradigm
c. Nursing students b. A theory
d. teaching c. A concept
d. A conceptual framework
Situation: you are actively practicing nurse who has just
finished you graduate studies. You learned the value of Situation: Mastery of research design determination is
research and would like to utilize the knowledge and essential in passing the NLE.
skills gained in the application of research to the nursing
service. The following question apply to research. 91. Ana wants to know if the length of time she will
study for the board examination is proportional
86. Which type of research inquiry investigates the to her board rating. During the June 2008 board
issues of human complexity (e.g. understanding examination, she studied for 6 months and
the human expertise) gained 68% , on the next board exam, she
a. Logical Position studied for 6 months again for a total of 1 year
b. Positivism and gained 74%, on the third board exam, she
c. Naturalistic inquiry studied for 6 months for a total of 1 and a half
d. Quantitative research
Year and gained 82%. The research design she Collected 100 random individuals and determine
used is: who is their favourite comedian actor. 50% said
a. Comparative Dolphy, 20% said Vic Sotto, while some answered
b. Experimental Joey De Leon, Allan K, Michael V. Tonyo conducted
c. Correlational what type of research study ?
d. Qualitative a. Phenomenological
b. Non experimental
92. Anton was always eating high fat diet. You want c. Case study
to determine if what will be the effect of high d. Survey
cholesterol food to Anton in the next 10 years.
You will use: 98. Jane visited tribe located somewhere in China, it
a. Comparative is called the Shin Jea tribe. She studied the way
b. historical of life, tradition and the societal structure of
c. correlational these people. Jane will best use which research
d. longitudinal design?
a. Historical
93. community A was selected randomly as well as b. Phenomenological
community B, nurse Edna conducted teaching to c. Case study
community A and assess if community A will d. ethnograpic
have a better status that community B. this is an
example of: 99. Anjoe researched on TB. Its transmission,
a. comparative causative agent and factors, treatment sign and
b. experimental symptoms as well as medication and all other in
c. correlational depth information about tuberculosis. This study
d. qualitative is best suited for which research design?
a. Historical
94. Ana researched on the development of a new b. Phenomenological
way to measure intelligence by creating a 100 c. Case study
item questionnaire that will assess the cognitive d. Ethnographic
skills of an individual. The design best suited for
this study is: 100. Diana is to conduct study about the relationship
a. Historical of the number of family members in the
b. Survey household and the electricity bill. Which of the
c. Methodological following is the best research design suited for
d. Case study this study?
1. Descriptive
95. Gen is conducting a research study on how mark, 2. Exploratory
an AIDS client lives his life. A design sited for this 3. Explanatory
study is: 4. Correlational
a. Historical 5. Comaparative
b. Phenomenological 6. Experimental
c. case study a. 1,4
d. ethnographic b. 2,5
c. 3,6
96. Marco is to perform a study about how nurses d. 1,5
perform surgical asepsis during World War II. A e. 2,4
design best for this study is:
a. Historical
b. Phenomenological
c. Case study
d. Ethnographic

97. Tonyo conducts sampling at barangay 412. He


TEST I- Foundation of Professional Nursing Practice 5. Nurse Betty is assigned to the following clients.
The client that the nurse would see first after
1. The nurse In- charge in labor and deliver unit endorsement?
administered a dose of terbutaline to a client a. A 34 year-old- post-operative
without checking the client’s pulse. The appendectomy client of five hours who is
standard that would be used to determine if the complaining of pain.
nurse was negligent is: b. A 44 year old myocardial infarction (MI)
a. The physician’s order client who is complaining of nausea
b. The action of a clinical nurse specialist who c. A 26 year old client admitted for
is recognized expert in the field dehydration whose intravenous (IV) has
c. The statement in the drug literature about infiltrated
administration of terbutaline d. A 63 year old post operative’s abdominal
d. The action of a reasonably prudent nurse hysterectomy client of three days whose
with similar education and experience. incisional dressing is saturated with
serosanguinous fluid.
2. Nurse Trish is caring for a female client with a
history of GI Bleeding, sickle cell disease, and a 6. Nurse Gail places a client in a four point
platelet count of 22,000/ul. The female client is restraint following orders from the physician.
dehydrated and receiving dextrose of 5% in haf The client care plan should include:
normal saline solution at 150 ml/hr. the client a. Assess temperature frequently
complains of severe bone pain and is scheduled b. Provide diversional activities
to receive a dose of morphine sulphate, In c. Check circulation every 15-30 minutes
administering the ,medication, Nurse Trish d. Socialize with other patients once a shift
should avoid which routine?
a. I.V 7. A male client who has severe burns is receiving
b. I.M H2 receptor antagonist therapy. The nurse in-
c. Oral charge knows the purpose of this therapy is to:
d. S.C a. Prevent stress ulcer
b. Block prostaglandin synthesis
3. Dr. Garcia writes the following order for the c. Facilitate protein synthesis
client who has been recently admitted d. Enhance gas exchange
“Digoxin .125 mg P.O. once daily.” To prevent a
dosage error, how should the nurse document 8. The doctors order hourly urine output
this order onto the medication administration measurement for a postoperative male client.
record? The nurse Trish records the following amounts
a. “Digoxin .1250 mg P.O. once daily.” of output for 2 consecutive hours: 8 a.m:50ml;
b. “Digoxin 0.1250 mg P.O. once daily.” 9a.m: 60ml. Based on these amounts, which
c. “Digoxin 0.125 mg P.O. once daily.” action should the nurse take?
d. “Digoxin .125 mg P.O. once daily.” a. Increase the I.V fluid infusion rate
b. Irrigate the indwelling urinary catheter
4. A newly admitted female client was diagnosed c. Notify the physician
with deep vein thrombosis. Which nursing d. Continue to monitor and record hourly
diagnosis should receive the highest priority? urine output
a. Ineffective peripheral tissue perfusion
related to venous congestion 9. Tony, a basketball player twist his right ankle
b. Risk for injury related to edema. while playing on the court and seeks care for
c. Excess fluid volume related to peripheral ankle pain and swelling. After the nurse applies
vascular disease ice to the ankles for 30minutes, which
d. Impaired gas exchange related to increased statement by Tony suggests that ice application
blood flow. has been effective?
a. “Many ankle look less swollen now”
b. “My ankle feels warm”
c. “My ankle appears redder now”
d. “I need something stringer for pain relief” d. Pulling the lobule down and forward

10. The physician prescribes a loop diuretic for a 16. Which instruction should nurse Tom give to a
client. When administering this drug, the nurse male client who is having external radiation
anticipates that the client may develop which therapy:
electrolyte imbalance? a. Protect the irritated skin from sunlight
a. Hypernatremia b. Eat 3 to 4 hours before treatment
b. Hyperkalemia c. Wash the skin over the regularly
c. Hypokalemia d. Apply lotion or oil to the radiated area
d. Hypervolemia when it is red or sore.
17. In assisting a female client for immediate
11. She finds out that some managers have surgery , the nurse in-charge is aware that she
benevolent-authoritative style of management. should:
Which of the following behaviours will she a. Encourage the client to avoid following
exhibit most likely? preoperative medication.
a. Have condescending trust and confidence in b. Explore the client’s fears and anxieties
their subordinates. about the surgery
b. Gives economic and ego awards c. Assist the client in removing dentures and
c. Communicates downwards to staffs nail polish
d. Allows decision making among d. Encourage the client drink water prior to
subordinates surgery

12. Nurse Amy is aware that the following is true 18. A male client is admitted and diagnosed with
about functional nursing acute pancreatitis after a holiday celebration of
a. Provides continuous, coordinated and excessive food and alcohol. Which assessment
comprehensive nursing activities finding reflects this diagnosis?
b. One to one nurse patient ratio a. Blood pressure above normal range
c. Emphasize the use of group collaboration b. Presence of crackles in both lung fields
d. Concentrates on tasks and activities c. Hyperactive bowel sounds
d. Sudden onset of continuous epigastric and
13. Which type of medication order might read back pain
“Vitamin K10 mg I.M daily x3 a day?”
a. Single order 19. Which dietary guidelines are important for
b. Standard written order Nurse Oliver to implement in caring for the
c. Standing order client with burns?
d. Stat order a. Provide high- fiber, high-fat diet
b. Provide high-protein, high- carbohydrate
14. A female client with a fecal Impaction diet
frequently exhibits which clinical manifestation? c. Monitor intake to prevent weight gain
a. Increased appetite d. Provide ice chips or water intake
b. Loss of urge to defecate
c. Hard, brown, formed stools 20. Nurse Hazel will administer a unit of whole
d. Liquid or semi-liquid stools blood, which priority information should the
nurse have about the client?
15. Nurse Linda prepares to perform an otoscopic a. Blood Pressure and pulse rate
examination on a female client. For proper b. Height and weight
visualization, the nurse should position the c. Calcium and potassium level
client’s ear by: d. Hgb and Hct Levels
a. Pulling the lobule down and back
b. Pulling the helix up and forward 21. Nurse Michelle witnesses a female client sustain
c. Pulling the helix up and back a fall and suspects that the leg may be broken.
The nurse takes which priority action?
a. Takes a set of vital signs
b. Call the radiology department for X-ray 27. A child of 10 years 400 cc of IV fluid in an 8 hour
c. Reassure the client that everything will be shift. The IV drip factor is 60. The IV rate that
alright will deliver ths amount is:
d. Immobilize the leg before moving the client a. 50 cc/ hour
b. 55 cc/ hour
22. A male client is being transferred to the nursing c. 24 cc/ hour
unit for admission after receiving a radium
implant for bladder cancer. The nurse-in charge 28. The nurse is aware that the most important
would take which priority action in the care of nursing action when a client returns surgery is:
this client? a. Assess the IV for type of fluid and rate of
a. Place client on reverse isolation flow.
b. Admit the client into the private room b. Assess the client for presence of pain
c. Encourage the client to take frequent rest c. Assess the foley catheter for patency and
periods urine output
d. Encourage family and friends to visit d. Assess the dressing for damage

23. A newly admitted female client was diagnosed 29. Which of the following vital sign assessment
with agranulocytosis. The nurse formulates that may indicate cardiogenic shock after
which priority nursing diagnosis? myocardial infraction?
a. Constipation a. BP- 80/60, Pulse – 110 irregular
b. Diarrhea b. BP- 90/50, Pulse – 50 regular
c. Risk of infection c. BP- 130/80, Pulse- 100 regular
d. Deficient knowledge d. BP- 180/100, Pulse- 90 irregular
24. A male client is receiving total perenteral
nutrition suddenly demonstrates signs and 30. Which is the most appropriate nursing action in
symptoms of an air embolism. What is the obtaining a blood pressure measurement
priority action by the nurse? a. Take the proper equipment, place the
a. Notify the physician client in the comfortable position, and
b. Place the client on the left side in the record the appropriate in information in
trendelenburg position the client’s chart
c. Place the client in high- fowlers position b. Measure the client’s arm, if you are not
d. Stop the total parenteral nutrition sure of the size of cuff to use.
c. Have the client recline or sit comfortably in
25. Nurse May attends an educational conference a chair with the forearm at the level of the
on leadership styles. The nurse is sitting with a heart.
nurse employed at a large trauma center who d. Document the measurement, which
states that the leadership style at the trauma extremity was used, and the position that
center is task-oriented and directive. The nurse the client was in during the measurement.
determines that the leadership style used at the
trauma center is: 31. Asking the questions to determine if the person
a. Autocratic understands the health teaching provided by
b. Laissez-faire the nurse would be included during which step
c. Democratic of the nursing process?
d. Situational a. Assessment
b. Evaluation
26. The physician orders DS 500 cc with KCI 10 c. Implementation
mEq/liter at 30cc/hr. the nurse in- charge is
going to hang a 500 cc bag. KCI will be added to
the IV solution.
a. .5cc
b. 5cc
c. 1.5cc
d. 2.5cc
d. Planning and goals “meperidine, 100mg/ml”. How many millilitres of
meperidine should the client receive?
32. Which of the following item is considered the a. 0.75
single most important factor in assisting the b. 0.6
health professional in arriving at a diagnosis or c. 0.5
determining the person’s need. d. 0.25
a. Diagnostic test result
b. Biographical date 38. A male client with diabetes mellitus is receiving
c. History of present illness insulin. Which statement correctly describes an
d. Physical examination insulin unit?
a. It’s a common measurement in metric
33. In preventing the development of an external system
rotation deformity of the hip in a client who b. It’s the basis for solids in the avoid drupois
must remain in bed for any period of time, the system
most appropriate nursing action would be to c. It’s the smallest measurement in the
use: apothecary system
a. Trochanter roll extending from the crest d. It’s a measurement of effect, not a standard
b. Pillows under the lower leg measure of weight or quantity.
c. Footboard
d. Hi-abductor pillow 39. Nurse Oliver measures a client’s temperature
102F. What is the equivalent centigrade
34. Which stage of pressure ulcer development temperature?
does the ulcer extend into the subcutaneous a. 40.1 degree Celsius
tissue? b. 38.9 degree Celsius
a. Stage I c. 48 degree Celsius
b. Stage II d. 38 degree Celsius
c. Stage III
d. Stage IV 40. The nurse is assessing a 48 year old client who
has come to the physician’s office for his annual
35. When the method of wound healing is one in physical exam. One of the first physical signs of
which wound edges are not surgically aging is:
approximated and integumentary continuity is a. Accepting limitations while developing
restored by granulations, the wound healing is assets
termed: b. Increasing loss of muscle tone.
a. Second intention healing c. Failing eyesight, especially close vision
b. Primary intention healing d. Having more frequent aches and pain
c. Third intention healing
d. First intention healing 41. A physician insert a chest tube into a female
client to treat a pneumothorax. The tube is
36. An 80 year old male client is admitted to the connected to water-seal drainage. The nurse in-
hospital with a diagnosis of pneumonia. Nurse charge can prevent chest tube air leaks by:
Oliver learns that the client live alone and a. Checking and taping all connections
hasn’t been eating or drinking. When assessing b. Checking patency of the test tube
him for dehydration, nurse Oliver would expect c. Keeping the head of the bed slightly
to find: elevated
a. Hypothermia d. Keeping the chest drainage system below
b. Hypertension the level of the chest
c. Distended neck veins
d. Tachycardia 42. Nurse Trish must verify the client’s identity
before administering medication. She is aware
37. The physician prescribes meperidine (Demerol), that the safest way to verify identity is to:
75mg I.M every 4 hours as needed, to control a a. Check the client’s identification band
client’s postoperative pain. The package insert b. Ask the client to state his name
c. State the client’s name out loud and wait a c. Every 2 years
client to repeat it d. Once, to establish baseline
d. Check the room number and the client’s
name on the bed 49. A male client has the following arterial blood
gas values: pH7.30;Pao2 89mmHg; Paco2 50
43. The physician orders dextrose 5% in water, mmHg; and HCO3 26mLq/L. Based on thses
1000 ml to be infused over 8 hours. The I.V values, Nurse Patricia should exoect which
tubing delivers 15drops/ml. Nurse John should condition?
run the I.V infusion at a rate of: a. Respiratory acidosis
a. 30 drops/minute b. Respiratory alkalosis
b. 32 drops/minute c. Metabolic acidosis
c. 20 drops/minute d. Metabolic alkalosis
d. 18 drops/minute
50. Nurse Len refers a female client with terminal
44. If a central venous catheter becomes cancer to a local hospice. What is the goal of
disconnected accidentally, what should the this referral?
nurse in-charge do immediately? a. To help the client find appropriate
a. Clamp the catheter treatment options
b. Call another nurse b. To provide support for the client and family
c. Call the physician in coping with termila illness
d. Apply a dry sterile dressing to the site. c. To ensure that the client gets counselling
regarding health care costs
45. A female client was recently admitted. She has d. To teach the client and family about cancer
fever, weight loss, and watery diarrhea is being and its treatment.
admitted to the facility. While assessing the
client, nurse hazel inspect the client’s abdomen 51. When caring for a male client with a 3 –cm
and notice that it is slightly concave. Additional stage I pressure ulcer on the coccyx, which of
assessment should proceed in which order? the following actions can the nurse institute
a. Palpitation, auscultation, and percussion independently?
b. Percussion, Palpitation and auscultation a. Massaging the area with an astringent every
c. Palpitation, percussion, and auscultation 2 hours
d. Auscultation, percussion and palpitation b. Applying an antibiotic cream to the area
three times a day
46. Nurse Betty is assessing tactile fremitus in a c. Using normal saline solution to clean the
client with pneumonia. For this examination, ulcer and applying protective dressing as
nurse Betty should use the: necessary
a. Fingertips d. Using a povidone-iodine wash on ulceration
b. Finger pads three times per day
c. Dorsal surface of the hand
d. Ulnar surface of the hand 52. Nurse Oliver must apply an elastic bandage to a
client’s ankle and calf. He should apply the
47. Which type of evaluation occurs continually bandage beginning Aat the client’s :
throughout the teaching and learning process? a. Knee
a. Summative b. Ankle
b. Informative c. Lower thigh
c. Formative d. Foot
d. Retrospective
53. A 10 year old child with type 1 diabetes develops
48. A 45 year old client, has no family history of diabetic ketoacidosis and receives a continuous
breast cancer or other risks factors for this insulin infusion. Which condition represents the
disease. Nurse John should instruct her to have greatest risk to this child?
mammogram how often? a. Hypernatremia
a. Twice per year b. Hypokalemia
b. Once per year c. Hyperphosphatemia
d. Hypercalcemia d. Obtaining the specimen from the urinary
drainage bag.
54. Nurse Len is administering sublingual
nitrglycerin (Nitrostat) to the newly admitted 59. Nurse Meredith is in process of giving a client a
afterward, the client may experience: bed bath. In the middle of the procedure, the
a. Throbbing headache or dizziness unit secretary calls the nurse on the intercom to
b. Nervousness or paresthesia tell the nurse that the there is an emergency
c. Drowsiness or blurred vision phone call. The appropriate nursing action is to:
d. Tinnitus or diplopia a. Immediately walk out of the client’s room
and answer the phone call
55. Nurse Michelle hears the alarm sound on the b. Cover the client, place the call light
telemetry monitor. The nurse quickly looks at c. Finish the bed bath before answering the
the monitor and notes that a client is in a phone call
ventricular tachycardia. The nurse rushes to the d. Leave the client’s door open so the client
client’s room. Upon reaching the client’s can be monitored and the nurse can answer
bedside, the nurse would take which action the phone
first?
a. Prepare for cardioversion 60. Nurse John is collecting a sputum specimen for
b. Prepare to defibrillate the client culture and sensitivity testing from a client
c. Call a code who has a productive cough. Nurse Janah plans
d. Check the client’s level of consciousness to implement which intervention to obtain the
specimen?
56. Nurse Hazel is preparing to ambulate a female a. Ask the client to expectorate a small
client. The best and safest position for the nurse amount of sputum into the emesis basin.
in assisting the client is to stand: b. Ask the client to obtain the specimen after
a. On the unaffected side of the client breakfast
b. On the affected side of the client c. Use a sterile plastic container for obtaining
c. In front of the client the specimen
d. Behind the client d. Provide tissues for expectoration and
obtaining the specimen
57. Nurse Janah is monitoring the ongoing care
given to the potential organ donor who has 61. Nurse Ron is observing a male client using a
been diagnosed with brain death. The nurse walker. The nurse determines that the client is
determines that the standard of acre had been using the walker correctly if the client:
maintained if which of the following data is a. Puts all the four points of the walker flat on
observed? the floor, puts weight on the hand pieces,
a. Urine output: 45 ml/hr and then walks into it.
b. Capillary refill: 5 seconds b. Puts weight on the hand pieces , moves the
c. Serum pH: 7.32 walker forward, and then walk into it
d. Blood pressure: 90/48 mmHg c. Puts weight on the hand pieces, slides the
walker forward, and then walks into it
58. Nurse Amy has an order to obtain a urinalysis d. Walks into the walker, puts weight on the
from a male client with an indwelling urinary hand pieces and then puts all four points of
catheter. The nurse avoids which of the the walker flat on the floor.
following. Which contaminate the specimen?
a. Wiping the port with an alcohol swab 62. Nurse Amy has documented an entry regarding
before inserting the syringe client care in the client’s medical record. When
b. Aspiring a sample from the port on the checking the entry, the nurse realizes that
drainage bag
c. Clamping the tubing of the drainage bag
Incorrect information was documented. How does the
nurse correct this error? a. Prone with head turned toward the side
a. Erases the error and writes in the correct supported by a pillow
information b. Sim’s position with the head of the bed
b. Uses correction fluid to cover up the incorrect flat
information and writes in the correct c. Right side-lying with the head of the bed
information. elevated 45 degrees.
c. Draws one line to cross out the incorrect d. Left side-lying with the head of the bed
information and then initials the change. elevated 45 degrees.
d. Covers up the incorrect information completely
using a black pen and writes in the correct 67. Nurse John develops methods for data
information gathering. Which of the following criteria of a
good instrument refers to the ability of the
63. Nurse Ron is assisting with transferring a client instrument to yield the same results upon its
from the operating room table to a stretcher. To repeated administration?
provide safety to the client , the nurse should: a. Validity
a. Moves the client rapidly from the table to b. Specificity
the stretcher c. Sensitivity
b. Uncover the client completely before d. Reliability
transferring to the stretcher.
c. Secures the client safety belts after 68. Harry knows that he has to protect the rights of
transferring to the stretcher human research subjects. Which of the following
d. Instructs the client to move self from the actions of Harry ensures anonymity?
table to the stretcher. a. Keep the identities to the subject secret
b. Obtain informed consent
64. Nurse Myrna is providing instruction to a c. Provide equal treatment to all the subjects
nursing assistant assigned to give a bed bath to of the study.
a client who is contact precautions. Nurse d. Release findings only to the participants of
Myrna instructs the nursing assistant to use the study.
which of the following protective items when
giving bed bath? 69. Patient’s refusal to divulge information is a
a. Gown and goggles limitation because it is beyond the control of
b. Gown and gloves Tiffany”. What type of research is appropriate
c. Gloves and shoe protectors for this study?
d. Gloves and goggles a. Descriptive- correlational
b. Experiment
65. Nurse Oliver is caring for a client with impaired c. Quasi- experiment
mobility that occurred as a result of a stroke. The d. Historical
client has right sided arm and leg weakness. The
nurse would suggest that the client use which of 70. Nurse Ronald is aware that the best tool for
the following assistive devices that would provide data gathering is?
the best stability and ambulating? a. Interview schedule
a. Crutches b. Questionnaire
b. Single and straight legged cane c. Use of laboratory data
c. Quad core d. Observation
d. Walker
71. Monica is aware that there are times when only
66. A male client with a right pleural effusion noted manipulation of study variables is possible and
on a chest X-ray is being prepared for the elements of control or randomization are
thoracentesis. The client experiences severe not attendant. Which type of research is
dizziness when sitting upright. To provide a safe referred to this?
environment, the nurse assists the client to a. Field study
which position for the procedure? b. Quasi- experiment
c. Solomon- four group design
d. Post-test only design d. Will remain unable to practice professional
nursing
72. Cherry notes down ideas that were derived
from the description of an investigation written 77. Ronald plans to conduct a research on the use
by the person who conducted it. Which type of of a new method of pain assessment scale.
reference source refers to this? Which of the following is the second step in the
a. Footnote conceptualizing phase of the research process?
b. Bibliography a. Formulating the research hypothesis
c. Primary source b. Review related literature
d. Endnotes c. Formulating and delimiting the research
problem
73. When nurse Trish is providing care to his d. Design the theoretical and conceptual
patient, she must remember that her duty is framework
bound no to do doing any action that will cause
the patient harm. Ti is the meaning of the 78. The leader of the study knows that certain
bioethical principle: patients who are in specialized research setting-
a. Non-malaficence tend to respond psychologically to the
b. Beneficence conditions of the study. This referred to as:
c. Justice a. Cause and effect
d. Solidarity b. Hawthorne effect
c. Halo effect
74. When a nurse in-charge causes an injury to a d. Horns effect
female and the injury caused becomes the
proof of the negligent act, the presence of the 79. Mary finally decides to use judgment sampling
injury is said to exemplify the principle of: on her research. Which of the following actions
a. Force majeure of is correct?
b. Respondeat superior a. Plans to include whoever is there during his
c. Res ipsa loquitor duty
d. Holdover doctrine b. Determines the different nationality of
patients frequently admitted and decides to
75. Nurse Myrna is aware that the Board of Nursing get representations samples from each.
has quasi-judicial power. An example of this c. Assigns numbers for each of the patients,
power is: place these in a fishbowl and draw 10 from
a. The Board can issue rules and regulation it.
that will govern the practice of nursing d. Decides to get 20 samples from the
b. The Board can investigate violations of the admitted patients.
nursing law and code of ethics
c. The Board can visit a school applying for a 80. The nursing theorist who developed
permit in collaboration with CHED transcultural nursing theory is:
d. The Board prepares the board a. Florence Nightingale
examinations. b. Madeleine Leininger
c. Albert Moore
76. When the license of nurse Krina is revoked, it d. Sr. Callista Roy
means that she:
a. Is no longer allowed to practice the 81. Marion is aware that the sampling method that
profession for the rest of her life gives equal chance to mall units in the
b. Will never have her/hi license re-issued population to get picked is:
since it has been revoked a. Random
c. May apply for re-issuance of his/her license b. Accidental
based on certain condition stipulated in RA c. Quota
9173 d. Judgment
d. Sim’s left lateral
82. John plans to use a Likert Scale to his study to 89. Nurse Marian is preparing to administer a blood
determine the: transfusion. Which action should the nurse take
a. Degree of agreement and disagreement first?
b. Compliance of expected standards a. Arrange for typing and cross matching of
c. Level of satisfaction client’s blood
d. Degree of acceptance b. Compare the client’s identification
wristband with the tag on the unit of blood.
83. Which of the following theory addresses the c. Start an I.V infusion of normal saline
four modes of adaption? d. Measure the client’s vital signs.
a. Madeleine Leininger
b. Sr. Callista Roy 90. A 65 year old male client request s his
c. Florence Nightingale medication at 9 p.m. instead of 10 p.m. so that
d. Jean Watson he can go to sleep earlier. Which type of nursing
intervention is required?
84. Ms. Garcia is responsible to the number of a. Independent
personnel reporting to her. This principle refers b. Dependent
to: c. Interdependent
a. Span of control d. Intradependent
b. Unity of command
c. Downward communication 91. A female client is to be discharged from an
d. Leader acute care facility after treatment for right leg
thrombophlebitis. The Nurse Betty notes that
85. Ensuring that there is an informed consent on the client’s leg is pain-free, without redness or
the part of the patient before a surgery is done, edema. The nurse’s actions reflect which step of
illustrates the bioethical principle of: the nursing process?
a. Beneficence a. Assessment
b. Autonomy b. Diagnosis
c. Veracity c. Implementation
d. Non-maleficence d. Evaluation

86. Nurse Reese is teacher a female client with 92. Nursing care for a female client includes
peripheral vascular disease about foot care; removing elastic stockings once per day. The
nurse Reese should include which instruction? Nurse Betty is aware that the rationale for this
a. Avoid wearing cotton socks. intervention?
b. Avoid using a nail clipper to cut toenails a. To increase blood flow to the heart
c. Avoid wearing canvas shoes. b. To observe the lower extremities
d. Avoid using cornstarch on feet. c. To allow the leg muscles to stretch and
relax
87. A client is admitted with multiple pressure d. To permit veins in the legs to fill with
ulcers. When developing the client’s diet plan, blood
the nurse should include:
a. Fresh orange slices 93. Which nursing intervention takes highest
b. Steamed broccoli priority when caring for a newly admitted client
c. Ice cream who’s receiving a blood transfusion?
d. Ground beef patties a. Instructing the client to report any itching,
swelling, or dyspnea
88. The nurse prepares to administer a cleansing b. Informing the client that the transfusion
enema. What is the most common client usually take 1 ½ to 2 hours.
position used for this procedure? c. Documenting blood administration in the
a. Lithotomy client care record.
b. Supine
c. Prone
d. Assessing the client’s vital signs hen the d. 30 minutes after administering the next
transfusions ends. dose.

94. A male client complains of abdominal 99. Nurse May is aware that the main advantage of
discomfort and nausea while receiving tube using a floor stock system is:
feedings. Which intervention is most a. The nurse can implement medication orders
appropriate for this problem? quickly.
a. Give the feedings at room temperature. b. The nurse receives input from the
b. Decrease the rate of feedings and the pharmacist.
concentration of the formula. c. The system minimizes transcription errors.
c. Place the client in semi-fowler’s position d. The system reinforces accurate circulations
while feeding.
d. Change the feeding container every 12 100. Nurse Oliver is assessing a client’s
hours. abdomen. Which finding should the nurse
report as abnormal?
95. Nurse Patricia is reconstituting a powdered a. Dullness over the liver.
medication in a vial. After adding the solution to b. Bowel sounds occurring every 10
the powder, she nurse should: seconds.
a. Do nothing c. Shifting dullness over the abdomen.
b. Invert the vial and let it stand for 3 to 5 d. Vascular sounds heard over the renal
minutes arteries.
c. Shake the vial vigorously.
d. Roll the vial gently between the palns.

96. Which intervention should the nurse Trish use


when administering oxygen by face mask to a
female client?
a. Secure the elastic band tightly around the
client’s head.
b. Assist the client to the semi-fowler position
if possible.
c. Apply the face mask from the client’s chin
up over the nose
d. Loosen the connectors between the oxygen
equipment and humidifier.

97. The maximum=m transfusion time for a unit of


packed red blood cells (RBCs) is:
a. 6 hours
b. 4 hours
c. 3 hours
d. 2 hours

98. Nurse Monique is monitoring the effectiveness


of a client’s drug therapy. When should the
nurse Monique obtain a blood sample to
measure the trough drug level?
a. 1 hour before administering the next dose
b. Immediately before administering the next
dose
c. Immediately after administering the next
dose
Answer and Rationale – Foundation of Professional Rationale: Curling’s ulcer occurs as a generalized stress
Nursing Practice response in burn patients. The results in a decreased
production of mucus and increased secretion of gastric
1. Answer: (D) the actions of a reasonably prudent acid. The best treatment for this prophylactic use of
nurse with similar education and experience. antacids and H2 receptor blockers.
Rationale: The standard of care is determined 8. Answer: (D) Continue to monitor and record
by the average degree of skill, care and hourly urine output
diligence by nurses in similar circumstances. Rationale: Normal urine output for an adult is
2. Answer: (B) I.M approximately 1 ml/minute (60ml/hour).
Rationale: With a platelet count of 22,000/ul, Therefore, this client’s output is normal. Beyond
the clients tends to bleed easily. Therefore, the continued evaluation, no nursing action is
nurse should avoid using the I.M. route because warranted.
the area is a highly vascular and can bleed 9. Answer: (B) “My ankle feels warm”.
readily when penetrated by a needle. The Rationale: Ice application decreases pain and
bleeding can be difficult to stop. swelling. Continued or increased pain, redness
3. Answer: (C) “Digoxin 0.125 mg P.O once daily” and increased warmth are signs of inflammation
Rationale: The nurse should always place a zero that shouldn’t occur after ice application.
before a decimal point so that no one misreads 10. Answer: (B) Hyperkalemia
the figure, which could result in a dosage error. Rationale: A loop diuretic removes water and,
The nurse should never insert a zero at the end along with it, sodium and potassium. This may
of a dosage that includes a decimal point result in hypokalemia, hypovolemia, and
because this could be misread, possibly leading hyponatremia.
to a tenfold increase in the dosage. 11. Answer: (A) Have condescending trust and
4. Answer: (A) Ineffective peripheral tissue confidence in their subordinates
perfusion related to venous congestion. Rationale: Benevolent-authoritative managers
Rationale: Ineffective peripheral tissue pretentiously show their trust and confidence
perfusion related to venous congestion takes to their followers
the highest priority because venous 12. Answer: (A) Provides continuous, coordinated
inflammation and clot formation impede blood and comprehensive nursing services.
flow in a client with deep vein thrombosis. Rationale: Functional nursing is focused tasks
5. Answer: (B) A 44 year-old myocardial infarction and activities and not on the care of the
(IM) client who is complaining of nausea. patients.
Rationale: Nausea is a symptom of impending 13. Answer: (B) Standard written order
myocardial infarction (MI) and should be Rationale: This is a standard written order.
assessed immediately so that treatment can be Prescribers write a single order for medications
instituted and further damage to the heart is given only once. A stat order is written for
avoided. medications given immediately for an urgent
6. Answer: (C) Check circulation every 15-30 client problem. A standing order, also known as
minutes. a protocol, establishes guidelines for treating a
Rationale: Restraints encircle the limbs, which particular disease or set of symptoms in special
place the client at risk for circulation every 15- care areas such as the coronary care unit.
30 minutes will allow the nurse to adjust the Facilities also may institute medication
restraints before injury from decreased blood protocols that specifically designate drugs that a
flow occurs. nurse may not give.
7. Answer: (A) Prevent stress ulcer 14. Answer: (D) Liquid or semi liquid stools
rationale: Passage of liquid or semi liquid stools
result from seepage of unformed bowel
contents around the impacted stool in the
return. Clients with fecal impaction don’t pass
hard, brown, formed stools because the feces
can’t move past the impaction. These
Clients typically report the urge to defecate Rationale: The client who has a radiation. Implant is
(although they can’t pass stool) and a decreased placed in a private room and has a limited number of
appetite. visitors. This reduces the exposure of others to the
15. Answer: ( C) Pulling the helix up and back radiation.
Rationale: to perform an otoscopic examination
on an adult, the nurse grasps the helix of the 23. Answer: ( C ) Risk for infection
ear and pulls it up and back to straighten the Rationale: Agranulocytosis is characterized by a
ear canal. Pulling the lobule in any direction reduced number of leukocytes (leucopenia) and
wouldn’t straighten the ear canal for neutrophils (neutropenia) in the blood. The
visualization. client is at high risk for infection because of the
16. Answer: (A) Protect the irritated skin from decreased body defences against
sunlight. microorganism. Deficient knowledge related to
Rationale: irradiated skin is very sensitive and the nature of the disorder may be appropriate
must be protected with clothing or sunblock. diagnosis but is not the priority.
The priority approach is the avoidance of strong 24. Answer: (B) Place the client on the left side in
sunlight. the tredelenburg position.
17. Answer: (C) Assist the client in removing Rationale: Lying on the left side may prevent air
dentures and nail polish. from flowing into the pulmonary veins. The
Rationale: Dentures, hairpins, and combs must trendelenburg position increases intrathoracic
be removed. Nail polish must be removed so pressure, which decreases the amount of blood
that cyanosis can be easily monitored by pulled into the vena cava during aspiration.
observing the nail beds. 25. Answer: (A) Autocratic
18. Answer: (D) sudden onset of continuous Rationale: The autocratic style of leadership is a
epigastric and back pain. task- oriented and directive.
Rationale: The auto digestion of tissue by the 26. Answer: (D) 2.5cc
pancreatic enzymes results in pain from Rationale: 2.5 cc is to be added, because 500
inflammation, edema, and possible bag of solution is being medicated instead of a 1
hemorrhagre. Continuous unrelieved epigastric liter.
or back pain reflects the inflammatory process 27. Answer: (A) 50cc/hour
in the pancreas. Rationale: A rate of 50cc/hr. The child is to
19. Answer: (B) provide high-protein, high- receive 400cc over a period of 8 hours =
carbohydrate diet. 50cc/hr.
Rationale: A positive nitrogen balance is 28. Answer: (B) Assess the client for presence of
important for meeting metabolic needs, tissue pain.
repair, and resistance to infection. Caloric goals Rationale: assessing the client for pain is a very
may be as high as 500 calories per day. important measure. Postoperative pain an
20. Answer: (A) Blood pressure and pulse rate. indication of complication. The nurse should
Rationale: The baseline must be established to also asses the client for pain to provide for the
recognize the signs of an anaphylactic or client’s comfort.
haemolytic reaction to the transfusion. 29. Ansewer: (A) BP- 80/60, Pulse- 110 irregular.
21. Answer: (D) immobilize the leg before moving Rationale: The classic signs of cardiogenic shock
the client. are low blood pressure, rapid and weak
Rationale: If the nurse suspects a fracture, irregular pulse, cold, clammy skin, decreased
splinting the area before moving the client is urinary output, and clebral hypoxia.
imperative. The nurse should call for emergency 30. Answer: (A) Take the proper equipment, place
help if the client is not hospitalized and call for a the client in a comfortable position. And record
physician for the hospitalized client. the appropriate information in the cleint’s
22. Answer: (B) Admit the client into a private chart.
room. Rationale: It is a general or comprehensive
statement about the correct procedure and it
indicates the basic ideas which are found in the
other options.
31. Answer: (B) Evaluation 43.

44.

32. No question

33. No question

34. No question

35. No question

36. No question

37. No question

38. No question

39. No question

40.

41.

42. Mnmnm 45.


46. Answer: (D) Ulnar Surface of the hand Interventions and will protect the area. Using
Rationale: The nurse uses the ulnar surface, or povidone-iodine wash and an antibiotic cream
ball of the hand to assess tactile fremitus, thrills require a physician’s order. Massaging with an
and vocal vibrations through the chest wall. The astringent can further damage the skin.
fingertips and finger pads best distinguish text 52. Answer: (D) Foot
and shape. The dorsal surface best feels Rationale: An elastic bandage should be applied
warmth. form the distal area to the proximal area. This
47. Answer: (C) Formative method promotes venous return. In this case,
Rationale: Formative (or concurrent) evaluation the nurse should begin applying the bandage at
occurs continuously throughout the teaching the client’s foot. Beginning at the ankle, lower
and learning process. One benefit is the nurse thigh or knee does not promote venous return.
can adjust teaching strategies as necessary to 53. Answer: (B) Hypokalemia
enhance learning. Summative or retrospective, Rationale: Insulin administration causes glucose
evaluation occurs at the conclusion of the and potassium to move into the cells, causing
teaching and learning session. Informative is not hypokalemia.
type of evaluation. 54. Answer: (A) Throbbing headache or dizziness
48. Answer: (B) Once per year Rationale: Headache and dizziness often occur
Rationale: yearly mammograms should begin at when nitroglycerin is taken at the beginning
age 40 and continue for as long as the woman is therapy. However, the client usually develops
in good health. If health risk, such as family tolerance.
history, genetic tendency or past breast cancer, 55. Answer: (D)Check the client’s level of
exist more frequent examinations may be consciousness
necessary. Rationale: Determining unresponsiveness is the
49. Answer: (B) Respiratory Acidosis first step assessment action to take. When a
Rationale: The client has a below-normal (acidic) client is in ventricular tachycardia, there is a
blood pH value and an above-normal partial significant decrease in cardiac output. However,
pressure of arterial carbon dioxide (Paco2) value, checking the unresponsiveness ensures
indicating respiratory acidosis. In respiratory whether the client is affected by the decreased
alkalosis, the pH value is below normal. In cardiac output.
metabolic acidosis, the pH and bicarbonate (Hco3) 56. Answer: (B) On the affected side of the client
values are below normal. In metabolic alkalosis, Rationale: When walking with clients, the nurse
the pH and Hco3 values are above normal.
should stand on the affected side and grasp the
50. Answer: (A) To provide support for the client and
security belt in the midspine area of the small of
family in coping with terminal illness.
the back. The nurse should position the free
Rationale: Hospices provide supportive care for
terminally ill clients and their families. Hospice hand at the shoulder area so that the client can
care doesn’t focus on counselling regarding health be pulled toward the nurse in the event that
care costs. Most client referred to hospices have there is a forward fall. The client is instructed to
been treated for their disease without success and look up and outward rather than at his or her
will receive only palliative care in the hospice. feet.
51. Answer: (C ) Using normal saline solution to clean 57. Answer: (A) Urine output: 45ml/hr
the ulcer and applying a protective dressing as Rationale: Adequate perfusion must be
necessary. maintained to all vital organs in order for the
Rationale: washing the area with normal saline client to remain visible as an organ donor. A
solution and applying a protective dressing are urine output of 45ml per hour indicates
within the nurse’s realm of adequate renal perfusion. Low blood pressure
and delayed capillary refill are circulatory
system indicators of inadequate perfusion. A
serum pH of 7.32 is acidotic, which adversely
affects all body tissues.
58. Answer: (D) Obtaining the specimen from the
urinary drainage bag.
Rationale: A urine specimen is not taken from the Because of the risk for potential heat loss.
urinalysis drainage bag. Urine undergoes chemical Hurried movements and rapid changes in the
changes while sitting in the bag and does not necessarily position should be avoided because these
reflect the current client status. In addition, it may predispose the client to hypotension. Sat the
become contaminated with bacteria from opening the time of the transfer from the surgery table to
system. the stretcher, the client is still affected by the
59. Answer: (B) cover the client, place the call light effects of the anaesthesia; therefore, the client
within reach, and answer the phone call. should not move self. Safety belts can prevent
Rationale: Because telephone call is an the client from falling off the stretcher.
emergency, the nurse may need to answer it. 64. Answer: (B) Gown and gloves
The other appropriate action is to ask another Rationale: Contact precautions require the use
nurse to accept the call. However, is not one of of gloves and a gown if direct client contact is
the options. To maintain privacy and safety, the anticipated. Googles are not necessary unless
nurse covers the client and places the call light the nurse anticipates the splashes of blood,
within the client’s reach. Additionally, the body fluids, secretions, or excretions may occur.
client’s door should be closed or the room Shoe protectors are not necessary
curtains pulled around the bathing area. 65. Answer: (C ) Quad Cane
60. Answer: (C) Use a sterile plastic container for Rationale: Crutches and a walker can be difficult
obtaining the specimen. to manuever for a client with weakness on one
Rationale: Sputum specimens for culture and side. A cane is better suited for client with
sensitivity testing need to be obtained using weakness of the arm and leg on one side.
sterile techniques because the test id done However, the Quad cane would provide the
determine the presence of organisms. If the most stability because of the structure of the
procedure for obtaining the specimen is not cane and because a quad cane ha sfour legs.
sterile, then the specimen is not sterile, then 66. Answer: (D) Left side lying with the head of the
the specimen would be contaminated and the bed elevated 45 degrees.
results of the test would be invalid. Rationale: To facilitate removal of fluid from the
61. Answer: (A) Puts all the four points of the chest wall, the client is poisoned sitting at the
walker flat on the floor, puts weight on the edge of the bed leaning over the bedside table
hand pieces , and then walks into it. with the feet supported on a stool. If the client
Rationale: When the client uses a walker, the is unable to sit up, the client is positioned lying
nurse stand adjacent to the affected side. The in bed on the unaffected side with the head of
client is instructed to put all four points of the the bed elevated 30 to 45 degrees.
walker to 2 feet forward flat on the floor before 67. Answer: (D) Reliability
putting weight on hand pieces. This will ensure Rationale: Reliability is consistency of the
client safety and prevent stress cracks in the research instrument. It refers to the
walker. The client is then instructed to move repeatability of the instrument in extracting the
the walker forward and walk into it. same responses upon its repeated
62. Answer: ( C) Draws one illness to cross out the administration.
incorrect information and then initials the 68. Answer: (A) Keep the identities of the subject
change. secret
Rationale: To correct an error documented in a Rationale: Keeping the identities of the research
medical record, the nurse draws one line subject secret will ensure anonymity because
through the incorrect information and then this will hinder providing link between the
initials the error. An error is never erased and information given to whoever is its source.
correction fluid is never used in the medical 69. Answer: (A) Descriptive-Correlational
record. Rationale: Descriptive-correlational study is the
63. Answer: ( C ) Secures the client safety belts after most appropriate for this study because it
transferring to the stretcher. studies the variables that could be the
Rationale: During the transfer the client after antecedents of the increased incidence of
the surgical procedure is complete. The nurse nosocomial infection.
should avoid exposure of the client
70. Answer: (C ) Use of laboratory data the working conditions of the workers on their
Rationale: Incidence of nosocomial infection is productivity. It resulted to an increased
best collected through the use of biophysiologic productivity but not due to the intervention but
measures, particularly in vitro measurements, due to the psychological effects of being
hence laboratory data is essential. observed. They performed differently because
71. Answer: (B) Quasi-Experimental they were under observation.
Rationale: Quasi-Experimental is done when 79. Answer: (B) Determines the different nationality
randomized and control of the variables are not of patients frequently admitted and decides to
possible. get representation samples from each.
72. Answer: (C ) Primary Source Rationale: Judgment sampling involves including
Rationale: This refers to a primary source which samples according to the knowledge of the
is a direct account of the investigation done by investigator about the participants in the study.
the investigator. In contrast to this is a 80. Answer: (B) Madeleine Leininger
secondary source, which is written by someone Rationale: Medeleine Leininger developed the
other than the original researcher. theory on transcultural theory based on her
73. Answer: (A) Non-maleficence observations on the behaviour of selected
Rationale: Non-maleficence means do not cause people within a culture.
harm or do any action that will cause any harm 81. Answer: (A) Random
to the patient/client. To do good is referred as Rationale: Random sampling gives equal chance
beneficence. for all the elements in the population to be
74. Answer: (C ) Res ipsa loquitor picked as part of the sample.
Rationale: Res ipsa loquitor literraly means the 82. Answer: (A) Degree of agreement and
thing speaks for itself. This means in operational disagreement
terms that the injury caused is the proof that Rationale: Likert Scale is a 5 point summated
there was a negligent act. scale used to determine the degree of
75. Answer: (B) The board can investigate violence agreement or disagreement of the respondents
of the nursing law and code of ethics to a statement in a study.
Rationale: Quasi-judicial power means that the 83. Answer: (B) Sr. Callista Roy
board of nursing has the authority to investigate Rationale: Sr. Callista Roy developed the
violations of the nursing law and can issue adaptation model which involves the
summons, subpoena, or subpoena duces physiologic mode, self-concept mode, role
tecumas needed. function mode and dependence mode.
76. Answer: ( C) May apply for re-issuance of 84. Answer: (A) Span of control
his/her license based on certain conditions Rationale: Span of control refers to the number
stipulated in RA 9173 of workers who report directly to a manager.
Rationale: RA 9173 sec. 24 states that for equity 85. Answer: (B) Autonomy
and justice, a revoked license maybe re-issued Rationale: Informed consent means that the
provided that the following conditions are met: patient fully understand about the surgery,
a) the cause for revocation of license has including the risks involved and the alternative
already been corrected or removed; and b) at solution. In giving consent it is done with
atleast four years has elapsed since the license knowledge and is given freely. The action of
has been revoked. allowing the patient to decide whether a
77. Answer: (B) Review related literature surgery is to be done or not exemplifies the
Rational: after formulating and delimiting the bioethical principle of anatomy
research problem, the researcher conducts a 86. Answer: (C) Avoid wearing canvas shoes.
review of a related literature to determine the Rationale: The client should be instructed to
extent of what has been done 0n the study by avoid wearing canvas shoes. Canvas shoes
previous researcher. cause the feet to perspire, which may, in turn,
78. Answer: (B) Hawthorne effect cause skin irritation and breakdown. Both
Rational: Hawthorne effect is based on the cotton and corn starch absorb perspiration.
study of Elton mayo and company about the
effect on an intervention done to improve
The client should be instructed to cut toenails Nursing process where the nurse puts the plan
straight across with nail clippers. to care into action.
87. Answer: (D) Ground beef patties 92. Answer: (B) To observe the lower extremities
Rationale: Meat is an excellent source of Rationale: Elastic stockings are used to promote
complete protein, which this client needs to venous return. The nurse needs to remove
repair the tissue breakdown caused by pressure them once per day to observe the condition of
ulcers. Oranges and broccoli supply vitamin C the skin underneath the stockings. Applying the
but not protein. Ice cream supplies only some stockings increases blood flow to the heart.
incomplete protein, making it less helpful in Where the stockings are in place, the leg
tissue repair. muscles can still stretch and relax, and the veins
88. Answer: (D) Sim’s left lateral can fill with blood.
Most common position used to administer a 93. Answer: (A) Instructing the client to report any
cleansing anema because it allows gravity to aid itching, swelling or dyspnea.
the flow of fluid along the curve of the sigmoid Rationale: Because administration of blood or
colon. If the client assume this position nor has blood products may cause serious adverse
poor sphincter control, the dorsal recumbent or effects such as allergic reactions, the nurse must
right lateral position may be used. The supine monitor the client for these effects. Signs and
and prone positions are inappropriate and symptoms of life-threatening allergic reaction
uncomfortable for the client. include itching, swelling, and dyspnea. Although
89. Answer: (A) Arrange for typing and cross the nurse should inform the client of the
matching of the clients blood. duration of the transfusion and should
Rationale: The nurse first arranges for typing document its administration, these actions are
and cross matching of the clients blood to less critical to the client’s immediate health. The
ensure compatibility with donor blood. The nurse should assess vital signs at least hourly
other options, although appropriate when during the transfusion.
preparing to administer a blood transfusion, 94. Answer: (B) Decrease the rate of feedings and
come later. the concentration of the formula.
90. Answer: (A) Independent Rationale: Complaints of abdominal discomfort
Rationale: Nursing interventions are classified and nausea are common in clients receiving
as independent, interdependent, or dependent. tube feedings. Decreasing the rate of the
Altering the drug schedule to coincide with the feeding and the concentration of the formula
client’s daily routine represents an indepedents should decrease the client’s discomfort.
intervention, whereas consulting with the Feedings are normally given at room
physician and pharmacist to change a client’s temperature to minimize abdominal cramping.
medication because of adverse reactions To prevent aspiration during feeding, the head
represents an interdependent intervention. of the client’s bed should be elevated at least
Administering an already-prescribed drug on 30 degrees. Also, to prevent bacterial growth,
time is a dependent intervention. An feeding containers should be routinely changed
intradependent nursing intervention doesn’t every 888 to 12 hours.
exist. 95. Answer: (D) Roll the vital gently between the
91. Answer: (D) Evaluation palms.
Rationale: The nursing actions described Rationale: Rolling the vital gently between the
constitute evaluation of the expected palms produces heat, which help dissolve the
outcomes. The finding shows that the expected medication. Doing nothing or inverting the vial
outcomes have been achieved. Assessment wouldn’t help dissolve the medication. Shaking
consist of a client’s history, physical the vial vigorously could cause the medication
examination, and laboratory studies. Analysis to breakdown, altering its action.
consist of considering assessment information 96. Answer: (B) Assist the client to the semi-fowler
to derive the appropriate nursing diagnosis. position if possible.
Implementation is the phase of the Rationale: by assisting the client to semi-fowler
position, the nurse promotes easier chest
expansion, breathing and oxygen intake.
The nurse should secure elastic band so that the
face mask fits comfortably and snugly rather
than tightly, which could lead to irritation. The
nurse should apply the face mask from the
client’s nose down to the chin- not vice versa.
The nurse should check the connectors
between the oxygen equipment and humidifier
to ensure that they are airtight; loosened
connectors can cause loss of oxygen.
97. Answer: (B) 4 hours
Rationale: A unit of packed RBCs may be giver
over a period of between 1 and 4 hours. It
shouldn’t refuse for longer than 4 hours
because the risk of contamination and sepsis
increases after that time. Discard or return to
the blood bank any blood not given within this
time, according to facility policy.
98. Answer: (B) immediately before administering
the next dose.
Rationale: Measuring the blood drug
concentration helps determine whether the
dosing has achieved the therapeutic goal. For
measurement of the trough, or lowest, blood
level of a drug, the nurse draws a blood sample
immediately before administering the next
dose. Depending on the drug’s duration of
action and half-life, peak blood drug levels
typically are drawn after administering the next
dose.
99. Answer: (A) The nurse can implement
medication orders quickly.
Rationale: A floor stock system enables the
nurse to implement medication orders quickly.
It doesn’t allow for pharmacist input, nor does it
minimize transcription errors or reinforce
accurate calculations.
100. Answer: (C ) Shifting dullness over the
abdomen
Rationale: Shifting dullness over the abdomen
indicates ascites, an abdominal finding. The
other options are normal abdominal findings.
TEST II- Community Health Nursing and Care of the a. Excessive fetal activity.
Mother and Child b. Larger than normal uterus for gestational
age
1. May arrives at the health care clinic and tells the c. Vaginal bleeding
nurse that her last menstrual period was 9 d. Elevated levels of human chorionic
weeks ago. She also tells the nurse that a home gonadotropin.
pregnancy test was positive but she began to
have mild cramps and is now having moderate 6. A pregnant client is receiving magnesium sulfate
vaginal bleeding. During the physical for severe pregnancy induced hypertension
examination of the client, the nurse notes that (PIH). The clinical findings that would warrant
May has a dilated cervix. The nurse determines use of the antidote, calcium gluconate is:
that May is experiencing which type of abortion? a. Urinary output 90cc in 2 hours.
a. Inevitable b. Absent patellar reflexes
b. Incomplete c. Rapid respiratory rate above 40/min.
c. Threatened d. Rapid rise in blood pressure
d. Septic
7. During vaginal examination of Janah who is in
2. Nurse Reese is reviewing the record of a labor, the presenting part is at station plus two.
pregnant client for her first prenatal visit. Which Nurse, correctly interprets it as:
of the following data, if noted on the client’s a. Presenting part is 2 cm above the plane of the
record, would alert the nurse that the client is ischial spines
at risk for a spontaneous abortion? b. Biparietal diameter is at the level of the
ischial spines
a. Age of 36 years
c. Presenting part in 2 cm below the plane of
b. History of syphilis
the ischial spines.
c. History of genital herpes
d. Biparietal diameter is 2 cm above the
d. History of diabetic mellitus
ischial spines.
3. Nurse Hazel is preparing to care for a client who
8. A pregnant client is receiving oxytocin (Pitocin)
is newly admitted to the hospital with a possible
for induction of labor. A condition that warrant
diagnosis of ectopic pregnancy. Nurse Hazel
the nurse in-charge to discontinue I.V infusion
develops a plan of care for the client and
of Pitocin is:
determines that which of the following nursing
a. Contractions every 1 ½ minutes lasting 70-
actions is the priority?
80 seconds.
a. Monitoring weight
b. Maternal temperature 101.2
b. Assessing for edema
c. Early decelerations in the fetal heart rate.
c. Monitoring apical pulse
d. Fetal heart rate baseline 140-160 bpm.
d. Monitoring temperature
9. Calcium gluconate is being administered to a
4. Nurse Oliver is teaching a diabetic pregnant
client with pregnancy induced hypertension
client about nutrition and insulin needs during
(PIH). A nursing action that must be initiated as
pregnancy. The nurse determines that the client
the plan of care throughout injection of the
understand dietary and insulin needs if the
drug is:
client states that the second half of pregnancy
a. Ventilator assistance
requires:
b. CVP readings
a. Decreased caloric intake
c. EKG tracings
b. Increased caloric intake
d. Continuous CPR
c. Decreased insulin
d. Increase insulin
10. A trial for vaginal delivery after an earlier
caesarian, would likely to be given to a gravida,
5. Nurse Michelle is assessing a 24 year old client
who had:
with a diagnosis of hydatidiform mole. She is
aware that one of the following us unassociated
with his condition?
a. First low transverse caesarian was for active Infant looks for it. The nurse is aware that estimated
herpes type 2 infections; vaginal culture at age of the infant would be:
39 weeks pregnancy was positive a. 6 months
b. First and second caesarian were for b. 4 months
cephalopelvic disproportion c. 8 months
c. First caesarian through a classic incision as a d. 10 months
result of severe fetal distress.
d. First low transverse caesarian was for 16. Which of the following is the most prominent
breech position. Fetus in this pregnancy is in feature to public health nursing?
a vertex presentation. a. It involves providing home care to sick
people who are not confined in the
11. Nurse Ryan is aware that the best initial hospital.
approach when trying to take a crying toddler’s b. Services are provided free of charge to
temperature is: people within the catchments area.
a. Talk to the mother first and then to the c. The public health nurse functions as part of
toddler a team providing a public health nursing
b. Bring extra help so it can be done quickly. services.
c. Encourage the mother to hold the child d. Public health nursing focuses on preventive,
d. Ignore the crying and screaming. not curative, services.

12. Baby Tina a 3months old infant just had a cleft 17. When the nurse determined whether resources
lip and palate repair. What should the nurse do were maximized in implementing Ligtas Tigdas,
to prevent trauma to operative site? she is evaluating
a. Avoid touching the suture line, even when a. Effectiveness
cleaning b. Efficiency
b. Place the baby in prone position. c. Adequacy
c. Give the baby a pacifier. d. Appropriateness
d. Place the infant’s arms in soft elbow
restraints. 18. Vangie is a new B.S.N. graduate. She wants to
become a Public Health Nurse. Where should
13. Which action should nurse Marian include in she apply?
the care plan for a 2 month old with heart a. Department of Health
failure? b. Provincial Health office
a. Feed the infant when he cries. c. Regional Health Unit
b. Allow the infant to rest before feeding. d. Rural Health Unit
c. Bathe the infant and administer
medications before feeding. 19. Tony is aware the Chairman of the Municipal
d. Weigh and bathe the infant before feeding. Health Board is:
a. Mayor
14. Nurse Hazel is teaching a mother who plans to b. Municipal Health Officer
discontinue breast feeding after 5months. The c. Public Health Nurse
nurse should advise her to include which foods d. Any qualified physician
in her infant’s diet?
a. Skim milk and baby food 20. Myra is the public health nurse in a municipality
b. Whole milk and baby food with a total population of about 20,000. There
c. Iron-rich formula only are 3 rural health midwives among the RHU
d. Iron-rich formula and baby food personnel. How many more midwife items will
the RHU need?
15. Mommy Linda is playing with her infant, who is a. 1
sitting securely alone on the floor of the clinic. b. 2
The mother hides a toy behind her back and the c. 3
d. The RHU does not need any more midwife
item.
21. According to freeman and Heinrich, community 26. The nurse is caring for a primigravid client in the
health nursing is a developmental service. labor and delivery area. Which condition would
Which of the following best illustrates this place the client at risk for disseminated
statement? intravascular coagulation (DIC)?
a. The community health nurse continuously a. Intrauterine fetal death.
develops himself personally and b. Placenta accreta.
professionally. c. Dysfunctional labor
b. Health education and community organizing d. Premature rupture of the membranes
are necessary in providing community
health services. 27. A fullterm client is in labor. Nurse Betty is aware
c. Community health nursing is intended that the fetal heart rate would be:
primarily for the health promotion and a. 80 to 100 beats/minute
prevention and treatment of disease b. 100 to 120 beats/minute
d. The goal of community health nursing is to c. 120 to 160 beats/minute
provide nursing services to people in their d. 160 to 180 beats/minute
own places of residence.
28. The skin in the diaper area of a 7 month old
22. Nurse Tina is aware that the disease declared infant is excoriated and red. Nurse Hazel should
through Presidential Proclamation No. 4 as a instruct the mother to:
target for eradication in the Philippines is? a. Change the diaper more often.
a. Poliomyelitis b. Apply talc powder with diaper changes.
b. Measles c. Wash the area vigorously with each diaper
c. Rabies change.
d. Neonatal tetanus d. Decrease the infant’s fluid intake to
decrease saturating diapers.
23. May knows that the step in community
organizing participation. What is the primary 29. Nurse Carla knows that the common cardiac
goal of community organizing? anomalies in the children with Down Syndrome
a. Integration (trisomy 21) is:
b. Community organization a. Atrial septal defect
c. Community study b. Pulmonic stenosis
d. Core group formation c. Ventricular septal defect
d. Endocardial cushion defect
24. Beth a public health nurse takes an active role
in community participation. What is the primary 30. Malou was diagnosed with severe preeclampsia
goal of community organizing? is now receiving I.V. magnesium sulfate. The
a. To educate the people regarding community adverse effects associated with magnesium
health problems sulfate is:
b. To mobilize the people to resolve community a. Anemia
health problems b. Decreased urine output
c. To maximize the community’s resources in c. Hyperreflexia
dealing with health problems. d. Increased respiratory rate
d. To maximize the community’s resources in
dealing with health problems. 31. A 23 year old client is having her menstrual
period every 2 weeks that last for 1 week. This
25. Tertiary prevention is needed in which stage of type of menstrual patter is best define by:
the natural history is disease? a. Menorrhagia
a. Pre-pathogenesis b. Metrorrhagia
b. Pathogenesis c. Dyspareunia
c. Prodromal d. Amenorrhea
d. Terminal
32. Jannah is admitted to the labor and delivery b. Dehydration and diarrhea
unit. The critical laboratory result for this client c. Bradycardia and hypotension
would be: d. Petechiae and hematuria
a. Oxygen saturation
b. Iron binding capacity 38. To evaluate a woman understands about the
c. Blood typing use of diaphragm for family planning. Nurse
d. Serum calcium Trish asks her to explain how she will use the
appliance. Which response indicates a need for
33. Nurse Gina is aware that the most common further health teaching?
condition found during the second-trimester of a. “I should check the diaphragm carefully for
pregnancy is: holes every time I sue it”
a. Metabolic alkalosis b. “I may need a different size of diaphragm if I
b. Respiratory acidosis gain or lose weight more than 20 pounds”
c. Mastitis c. “The diaphragm must be left in place for
d. Physiologic anemia atleast 6 hours after intercourse”
d. “I really need to use the diaphragm and jelly
34. Nurse Lynette is working in the triage area of an most during the middle of my menstrual
emergency department. She sees that several cycle.”
pediatric clients arrive simultaneously. The
client who needs to be treated first is: 39. Hypoxia is a common complication of
a. A crying 5 year old child with a laceration on laryngotracheobronchitis. Nurse Oliver should
his scalp. frequently asses a child with
b. A 4 year old child with barking coughs and laryngotracheobronchitis for:
flushed appearance. a. Drooling
c. A 3 year old child with Down Syndrome who b. Muffled voice
is pale and asleep in his mother’s arms c. Restlessness
d. A 2 year old infant with stridorous breath d. Low-grade fever
sounds, sitting up in his mother’s arms and
drooling. 40. How should Nurse Michelle guide a child who is
blind to walk to the playroom?
35. Maureen in her third trimester arrives at the a. Without touching the child, talk
emergency room with painless vaginal bleeding. continuously as the child walks down the
Which of the following conditions is suspected? hall.
a. Placenta previa b. Walk one step ahead, with the child’s hand
b. Abruption placentae on the nurse’s elbow.
c. Premature labor c. Walk slightly behind, gently guiding the
d. Sexually transmitted disease child forward.
d. Walk next to the child, holding the child’s
36. A young child named Richard is suspected of hand.
having pinworms. The community nurse collects
a stool specimen to confirm the diagnosis. The 41. When assessing newborn diagnosed with
nurse should schedule the collection of this ductus arteriosus, Nurse Olivia should expect
specimen for: that the child most likely would have an:
a. Just before bedtime a. Loud, machinery-like murmur.
b. After the child has been bathe b. Bluish color to the lips.
c. Any time during the day c. Decreased BP reading in the upper
d. Early in the morning extremities.
d. Increased BP reading in the upper
37. In doing a child’s admission assessment, Nurse extremities.
Betty should be alert to note which signs or
symptoms of chronic lead poisoning? 42. The reason nurse May keeps the neonate in a
a. Irritability and seizures neutral thermal environment is that when a
47. Barangay Pinoy had an outbreak of German
newborn becomes too cool, the neonate requires measles. To prevent congenital rubella, what is
a. Less oxygen, and the newborn’s metabolic rate the BEST advice that you can give to women in
increases. the first trimester of pregnancy in the barangay
b. More oxygen, and the newborn’s metabolic Pinoy?
rate decreases. a. Advise them on the signs of German
c. More oxygen, and the newborn’s metabolic measles.
rate increases. b. Avoid crowded places, such as markets and
d. Less oxygen, and the newborn’s metabolic rate movie houses.
decreases. c. Consult at the health center where rubella
vaccine may be given
43. Before adding potassium to an infant’s I.V. line, d. Consult a physician who may give them
Nurse Ron must be sure to assess whether this rubella immunoglobulin.
infant has:
a. Stable blood pressure 48. Myrna a public health nurse knows that to
b. Patant frontanelles determine possible sources of sexually
c. Moro’s reflex transmitted infections, the BEST method that
d. Voided may be undertaken is:
a. Contact tracing
44. Nurse Carla should know that the most b. Community survey
common causative factor or dermatitis in c. Mass screening tests
infants and younger children is: d. Interview of suspects
a. Baby oil
b. Baby lotion 49. A 33-year old female client came for
c. Laundry detergent consultation at the health center with the chief
d. Powder with cornstarch complaint of fever for a week. Accompanying
symptoms were muscle pains and body malaise.
45. During tube feeding, how far above an infant’s A week after the start of fever, the client noted
stomach should the nurse hold the syringe with yellowish discoloration of his sclera. History
formula? showed that he waded in flood waters about 2
a. 6 inches weeks before the onset of symptoms. Based on
b. 12 inches her history, which disease condition will you
c. 18 inches suspect?
d. 24 inches a. Hepatitis A
b. Hepatitis B
46. In a mother’s class, Nurse Lynnete discussed c. Tetanus
childhood diseases such as chicken pox. Which d. Leptospirosis
of the following statements about chicken pox
is correct? 50. Mickey a 3-year old client was brought to the
a. The older one gets, the more susceptible he health center with chief complaint of severe
becomes to the complications of chicken diarrhea and the passage of “rice water” stools.
pox. The client is most probably suffering from which
b. A single attack of chicken pox will prevent condition?
future episode, including conditions such as a. Giardiasis
shingles. b. Cholera
c. To prevent an outbreak in the community, c. Amebiasis
quarantine may be imposed by health d. Dysentery
authorities.
d. Chicken pox vaccine is best given when 51. The most prevalent form of meningitis among
there is impending outbreak in the children aged 2 months to 3 years is caused by
community. which microorganism?
a. Hemophilus influenzea
b. Morbillivirus
c. Streptococcus pnuemoniae d. Use of p[protective footwear, such as
d. Neisseria meningitides rubber boots

52. The student nurse is aware that the 58. Several clients is newly admitted and diagnosed
pathognomonic sign of measles is Koplik’s spot with leprosy. Which of the following clients
and you may see Koplik’s spot by inspecting the: should be classified as a case of multibacillary
a. Nasal mucosa leprosy?
b. Buccal mucosa a. 3 skin lesion, negative slit skin smear
c. Skin on the abdomen b. 3 skin lesions, positive slit skin smear
d. Skin on neck c. 5 skin lesions, negative slit skin smear
d. 5 skin lesions, positive slit skin smear
53. Angel was diagnosed as having Dengue fever.
You will say that there is slow capillary refill 59. Nurses are aware that diagnosis of leprosy is
when the color of the nailbed that you pressed highly dependent on recognition of symptoms.
does not return within how many seconds? Which of the following Is an early sign of
a. 3 seconds leprosy?
b. 6 seconds a. Mascular lesions
c. 9 seconds b. Inability to close eyelids
d. 10 seconds c. Thickened painful nerves
d. Sinking of the nosebridge
54. In Integrated Management of Childhood Illness,
the nurse is aware that the severe conditions 60. Marie brought her 10 month old infant for
generally require urgent referral to a hospital. consultation because of fever, started 4 days
Which of the following severe conditions DOES prior to consultation. In determining malaria
NOT always require urgent referral to a hospital? risk, what will you do?
a. Mastoiditis a. Perform a tourniquet test.
b. Severe dehydration b. Ask where the family resides
c. Severe pneumonia c. Get a specimen for blood smear.
d. Sever febrile disease d. Ask if the fever is present every day.

55. Myrna a public health nurse is will conduct 61. Susie brought her 4 years old daughter to the
outreach immunization in a barangay Masay RHU because of cough and colds. Following the
with a population of about 1500. The estimated IMCI assessment guide, which of the following is
number of infants in the barangay would be: a danger sign that indicates the need for urgent
a. 40 infants referral to a hospital?
b. 50 infants a. Inability to drink
c. 55 infants b. High grade fever
d. 65 infants c. Signs of severe dehydration
d. Cough for more than 30 days
56. The community nurse is aware that the
biological used in Expanded Program on 62. Jimmy a 2-year old child revealed “baggy
Immunization (EPI) should NOT be stored in the pants”. As a nurse, using the IMCI guidelines,
freezer? how will you manage jimmy?
a. DPT a. Refer the child urgently to a hospital for
b. Oral polio vaccine confinement.
c. Measles vaccine b. Coordinate with the social worker to enroll
d. MMR the child in a feeding program.
c. Make a teaching plan for the mother,
57. It is the most effective way of controlling focusing on the menu planning for her child.
schistosomiasis in an endemic area? d. Assess and treat the child for health
a. Use of molluscicides problems like infections and intestinal
b. Building of foot bridges parasitism.
c. Proper use of sanitary tolilets
63. Gina is using Oresol in the management of 68. The nurse explains to a breastfeeding mother
diarrhea of her 3-year old child. She asked you that breast milk is sufficient for all the baby’s
what to do if her child vomits. As a nurse you nutrient need only up to:
will tell her to: a. 5 months
a. Bring the child to the nearest hospital for b. 6 months
further assessment. c. 1 year
b. Bring the child to the health center for d. 2 years
intravenous fluid therapy.
c. Bring the child to the health center for 69. Nurse Ron is aware that the gestational age of a
assessment by the physician. conceptus that is considered viable (able to live
d. Let the child rest for 10 minutes then outside the womb) is:
continue giving Oresol more slowly. a. 8 weeks
b. 12 weeks
64. Nikki a 5-month old infant was brought by his c. 24 weeks
mother to the health center because of diarrhea d. 32 weeks
for 4 to 5 times a day. Her skin goes back slowly
after a skin pinch and her eyes are sunken. 70. When teaching parents of a neonate the proper
Using the IMCI guidelines, you will classify this position for the neonate’s sleep, the nurse
infant in which category? Patricia stresses the importance of placing the
a. No signs of dehydration neonate on his back to reduce the risk of which
b. Some dehydration of the following?
c. Severe dehydration a. Aspiration
d. The data is insufficient. b. Sudden infant death syndrome (SIDS)
c. Suffocation
65. Chris a 4-month old infant was brought by her d. Gastroesophageal reflux (GER)
mother to the health center because of cough.
His respiratory rate is 42/minute. Using the 71. Which finding might be seen in baby James a
Integrated Management of Child Illness (IMCI) neonate suspected of having infection?
guidelines of assessment, his breathing is a. Flushed cheeks
considered as: b. Increased temperature
a. Fast c. Decreased temperature
b. Slow d. Increased activity level
c. normal
d. insignificant 72. Baby jenny who is small-for-gestation is at
increased risk during the transitional period for
66. Mylene had just received her 4th dose of tetanus which complication?
toxoid. She is aware that her baby will have a. Anemia probably due to chronic fetal
protection against tetanus for hyposia
a. 1 year b. Hyperthermia due to decreased glycogen
b. 3 years stores
c. 5 years c. Hyperglycemia due to decreased glycogen
d. Lifetime stores
d. Polycythemia probably due to chronic fetal
67. Nurse Ron is aware that unused BCG should be hypoxia
discarded after how many hours of
reconstitution? 73. Marjorie has just given birth at 42 weeks’
a. 2 hours gestation. When the nurse assessing the
b. 4 hours neonate, which physical finding is expected?
c. 8 hours a. A sleepy, lethargic baby
d. At the end of the day b. Lanugo covering the body
c. Desquamation of the epidermis
d. Vernix caseosa covering the body
74. After reviewing the Myrna’s maternal history of b. The parent’s expression if interest about
magnesium sulfate during labor, which the size of the new born.
condition would nurse Richard anticipate as a c. The parents’ indication that they want to
potential problem in the neonate? see the newborn.
a. Hypoglycemia d. The parents’ interaction with each other.
b. Jitteriness
80. Following a precipitous delivery, examination of
c. Respiratory depression
the client’s vagina reveals a fourth-degree
d. Tachycardia
laceration. Which of the following would be
75. Which symptom would indicate the Baby contraindicated when caring for this client?
Alexandra was adapting appropriately to extra- a. Applying cold to limit edema during the first
uterine life without difficulty? 12 to 24 hours.
a. Nasal flaring b. Instructing the client to use two or more
b. Light audible grunting
peripads to cushion the area.
c. Keep the cord dry and open air
d. Wash the cord with soap and water each c. Instructing the client on the use of sitz bath
day during a tub bath if ordered.
d. Instructing the client about the importance
76. When teaching umbilical cord care for Jennifer of perineal (kegel) exercise.
a new mother, the nurse Jenny would include 81. A pregnant woman accompanied by her
which information? husband, seeks and admission to the labor and
a. Apply peroxide to end the cord with each
delivery area. She states that she’s in labor and
diaper change
b. Cover the cord with petroleum jelly after says she attended the facility clinic for prenatal
bathing care. Which question should the nurse Oliver
c. Keep the cord dry and open to air ask her first?
d. Wash the cord with soap and water each a. “Do you have any chronic illness?”
day during a tub bath. b. “Do you have any allergies?”
c. “What is your expected due date?”
77. Nurse John is performing an assessment on a
d. “Who will be with you during labor?”
neonate. Which of the following findings is
considered common in the healthy neonate? 82. A neonate begins to gag and turns a dusky
a. Simian crease color, what should the nurse do first?
b. Conjunctival hemorrhage a. Calm the neonate
c. Cystic hygroma b. Notify the physician
d. Bulging fontanelle c. Provide oxygen via face mask as ordered
d. Aspirate the neonate’s nose and mouth
78. Dr. Esteves decides to artificially rupture the
membranes of a mother who is on labor. with a bulb syringe.
Following this procedure, the nurse hazel 83. When a client states that her “water broke,”
checks the fetal heart tones for which of the which of the following actions would be
following reasons? inappropriate for the nurse to do?
a. To determine fetal well-being a. Observing the pooling of straw-colored
b. To assests for prolapsed cord fluid.
c. To assess fetal position
b. Checking vaginal discharged with nitrazine
d. To prepare for an imminent delivery.
paper
79. Which of the following would be least likely to c. Conducting a bedside ultrasound for an
indicate anticipated bonding behaviors by new amniotic fluid index.
parents? d. Observing for flakes of vernix in the vaginal
a. The parent’s willingness to touch and hold discharge.
the new born.
84. A baby girl is born 8 weeks premature. At birth
she has no spontaneous respiration but is
successfully resuscitated. With several hours
she develops respiratory grunting, cyanosis,
tachypnea,nasal falring, and retractions. She’s
diagnosed with respiratory distress syndrome,
intubated, and placed on a ventilator. Which
nursing action should be include in the baby’s
plan of care to prevent retinopathy of
prematurity?
a. Cover his eyes while receiving oxygen.
b. Keep her body temperature low.
c. Monitor partial pressure of oxygen (pao2)
levels.
d. Humidify the oxygen.

85. Which of the following is normal newborn


calorie intake?
a. 110 to 130 calories per kg.
b. 30 to 40 calories per lb of body weight.
c. Atleast 2 ml per feeding
d. 90 to 1oo calories per kg.

86. Nurse John is knowledgeable that usually


individual twins will grow appropriately and at
the same rate as singletons until how many
weeks?
a. 16 to 18 weeks
b. 18 to 22 weeks
c. 30 to 32 weeks
d. 38 to 40 weeks
87. Which of the following classification applies to
monozygotic twins for whom the cleavage of
the fertilized ovum occurs more than 13 days
after fertilization?
a. Conjoined twins
b. Diamniotic dichorionic twins
c. Diamniotic monochorionic twins
d. Monoamniotic monochorionic twins

88.

You might also like